Космос и астрономия


Ответить в тред Ответить в тред

Check this out!
<<
Назад | Вниз | Каталог | Обновить тред | Автообновление
516 59 145

Тред тупых вопросов №107 MESSIER EDITION Аноним 22/05/19 Срд 13:48:03 4849081
CharlesMessier.jpg (53Кб, 621x787)
621x787
Catalog-1774.jpg (604Кб, 644x800)
644x800
MessierObjects.jpg (3052Кб, 3600x2197)
3600x2197
M87zoom.webm (4166Кб, 1280x720, 00:00:54)
1280x720
Тред вопросов о жизни, Вселенной и всём таком.

Спрашиваем то, за что в других местах выдают путёвку в биореактор. Здесь анонимные учёные мирового уровня критически рассмотрят любые гениальные идеи и нарисованные в Paint схемы.

Предыдущий тут: >>480719 (OP)
https://2ch.hk/spc/res/480719.html

Q: Можно быстрее?
A: Можно упасть в пузырь альбукерке, наса уже почти надула его.

Q: Я начитался охуительных историй про уфологию, че делать, нам жопа?
A: Да, тебе жопа, можешь сгонять в зогач или куда оттуда пошлют.

Q: Что будет с человеком в вакууме без скафандра / если он упадет на черную дыру / попробует ступить на поверхность газового гиганта/солнца?
A: Он умрёт.

Q: Почему бы не привязать ракету к воздушному шару или стартовать с горы?
A: Космос - это не как высоко, а как быстро, большая часть энергии ракеты уходит на разгон вбок.
Подробнее тут https://what-if.xkcd.com/58/ (английский) https://chtoes.li/orbital-speed/ (перевод)
Аноним 22/05/19 Срд 14:15:56 4849122
Ебанный стыд...
Во-первых, Алькубьерре.
Во-вторых, не упасть, а создавать вокруг корабля изнутри (иначе кина не будет).
В-третьих, НАСА искривляет пространство на десятимиллионную часть, контролируя это сверхточными интерферометрами, до самого варп-привода здесь - как до Антарктиды раком.
Аноним 22/05/19 Срд 14:30:53 4849173
>>484912
Когда эту пасту уже интегрируют в шапку?
Аноним 22/05/19 Срд 14:31:43 4849184
>>484917
Ее суть как раз в том, что она ответ на шапку.
Была еще одна паста отвечающая на эту пасту, и краткожившая паста ответ на пасту отвечавшую на пасту.
Аноним 22/05/19 Срд 14:32:54 4849195
>>484908 (OP)
>M87zoom.webm
А почему это очко саурона вейпает только в одну сторону?
Аноним 22/05/19 Срд 14:48:10 4849206
>>484908 (OP)
Для искусственной гравитации на космическом корабле нужно открыть (выделить) гравитон и направить гравитоны на пол корабля?
Аноним 22/05/19 Срд 14:49:51 4849217
Аноним 22/05/19 Срд 14:50:29 4849228
>>484920
Пока неясно, есть ли частицы отвечающие за силу притяжения, есть мнение, что это суть метрики пространства-времени, ее искажение.
В пользу этого говорит принцип эквивалентности ускорения.
Так что отвечая на твой вопрос: можно вообще ничего не выделять и просто ускоряться.
А если солярки мало - сделай центрифугу.
Аноним 22/05/19 Срд 14:51:15 4849239
>>484921
Ааа, из-за скорости плюм в другую сторону светит в другом спектре получается?
Аноним 22/05/19 Срд 14:52:47 48492410
>>484922
То есть достаточно сделать в космосе эффект центробежных сил?
Аноним 22/05/19 Срд 14:53:18 48492511
>>484919
Почему только в одну? В другую сторону хуярит не менее мощный джет, просто его с Земли не видно.

Из-за релятивистских эффектов движущееся в нашу сторону вещество кажется более ярким, а удаляющееся от нас — более тусклым, в случае с М87 просто разница в яркости очень большая из-за околосветовой скорости струи и направленности ее практически на Землю.
Аноним 22/05/19 Срд 16:23:43 48493012
Анон, посоветуй сайфая годного (сериалы, кинцо, книги, похуй). Главный критерий, чтобы все было реалистично (орбитальная механика, физика, технологии, мир и т. д.), но не слишком камерное, чтобы хотя бы в больших масштабах, чем лагерь на марсе.
Аноним 22/05/19 Срд 16:26:43 48493113
>>484930
Игры в этом разделе и так обсуждаются (кроме инженеров, они не про механику, это просто копрокубач): KSP, Space Engine, Orbiter
Кинцо: Apollo 13, First Man, Gravity... дополните дальше, в голову не приходит
Книги: дохуища. Лезешь в любимую библиотеку и выбираешь жанр Hard Sci-Fi, после чего выбираешь понравившуюся завязку.
Анимцо Planetes, кстати, грамотно космос показали. Если не против еще драмы-отношача заодно навернуть - понравится.
Аноним 22/05/19 Срд 16:26:58 48493214
>>484930
Почему не в sf? Средний обыватель там не до конца в состоянии оценить реалистичность мира. Нужна рекомендация ученых спейсачеров.
Аноним 22/05/19 Срд 16:28:09 48493315
Аноним 22/05/19 Срд 21:00:49 48495016
Если бы наша планета была как Кербин (3 тысячи дельты до орбиты при той же массе), кто бы первый добрался до космоса? Нацисты во вторую мировую, ребята типа Годдарда в начале XX века, китайцы в средние века?
Аноним 22/05/19 Срд 21:37:53 48495417
>>484950
Очень трудно рассуждать в сослагательном ключе, особенно учитывая что например у тех же римлян были все возможности и потребности запилить телеграфную сеть, и они даже знали о светосигнальных башнях от греков, но так и не запилили.

Правильный ответ - да хуй его знает, как поперло бы так и вышло бы.
Аноним 22/05/19 Срд 21:39:57 48495518
чё ам по плоскостям?))))))
Аноним 22/05/19 Срд 22:43:12 48497219
>>484950
Думаю ребята типа Годдарда, так как для создания орбитальных ракет не требовались бы серьезные технологические достижения и индустриальные мощности
Аноним 22/05/19 Срд 22:44:23 48497420
>>484958

Где-то проебался с порядками, должно было получиться 180 раз, а не 1800.
Аноним 23/05/19 Чтв 00:31:10 48499321
>>484950
Что за дельта, нахуй? Поясните.
Аноним 23/05/19 Чтв 06:13:19 48500122
>>484993
Характеристическая скорость/Δv/дельта-в. Максимальная скорость, до которой может разогнаться реактивный аппарат в вакууме при отсутствии других сил, от полного бака до пустого. Она же - универсальная мера доступной энергетики аппарата, независимо от его размеров. А также универсальная мера стоимости любого перехода между двумя орбитами, независимо от характеристик аппарата. Напрямую связана с мощностью двигателя и его удельным импульсом (экономичностью).

Это базовая хуйня в спейсаче. Если такие вопросы приходится задавать, читни любой учебник по орбитальной механике (Левантовского например), поиграй в любой симулятор - KSP, Orbiter, или вообще GMAT.
23/05/19 Чтв 10:12:00 48503423
Аноним 23/05/19 Чтв 10:21:02 48503624
>>485034
Алё, это тред тупых вопросов, он для этого и нужен.
Аноним 23/05/19 Чтв 10:22:07 48503725
>>484931
>Если не против еще драмы-отношача заодно навернуть - понравится.
Манга в этом деле лучше, там романтики нет.
Аноним 23/05/19 Чтв 11:09:24 48504226
15237172676340.jpg (36Кб, 374x370)
374x370
>>485034
Поссал тебе в ротеш, пидор

Автор вопроса про всплывающие пельмени
Аноним 23/05/19 Чтв 13:14:30 48504827
>>485034
Анон на спейсаче жесток. Сначала посылает в ТТВ, а оттуда посылает нахуй.
Аноним 23/05/19 Чтв 16:21:24 48510028
>>484958
А, так ты про градиент хотел сказать, так бы и сказал.
Аноним 23/05/19 Чтв 19:50:42 48511029
Почему не юзают ГПВРД при выводе космических аппаратов на низкую орбиту?
Аноним 23/05/19 Чтв 19:58:38 48511230
>>485110
Потому что твоя мамаша шлюха.
Аноним 23/05/19 Чтв 22:25:30 48515731
>>485110
Потому что их не существует в юзабельном виде.
Аноним 24/05/19 Птн 00:03:56 48517432
>>485110
Потому что проскочить атмосферу на порядок проще, чем ебаться с ней. Летать на гиперзвуке тяжело, греется пиздос. Кроме того, устойчивый гиперзвуковой полет это область малоисследованная, и востребованная в основном вояками. Входить в плотные слои на большой скорости под большими улами атаки гораздо легче, чем взлетать в них на этой скорости.

А так - есть полтора маняпроекта. Скайлон, которому скоро уже лет 35 будет как он на бумаге. И какой-то китайский стартапер с сумрачной вундервафлей, глянь в китаетреде. У Скайлона есть лишь концепт движка, за гиперзвуковую тушку они даже и не брались. У китайского школостартапера якобы какая-то хитровыебанная аэротермодинамическая схема, про которую нихуя не понятно.

Алсо, движком скайлона интересовалась дарпа и вроде как года 4 назад что-то там финансировали помимо viability study. Может у них там полноценный блэк проджект, а может хуй с маслом.
Аноним 24/05/19 Птн 00:41:38 48517833
>>485174
>Алсо, движком скайлона интересовалась дарпа и вроде как года 4 назад что-то там финансировали
Вот вы тут сидите, а пиндосы скоро втихаря колонизируют Альдебаран и ни с кем не поделятся.
Аноним 24/05/19 Птн 01:26:31 48518034
>>485178
Шутки шутками, а воякам наверно такое гибридное двигло очень кстати. Всякие летающие стелс табуретки в блэк режиме и делались и эксплуатировались в свое время. А сейчас вояки трех стран спят и видят гиперзвуковые ёбы, и гиперзвуковые тушки пока что существуют только у них, так что дарпа вполне могла что-нибудь нахимичить в очередном сканк ворксе. а могла и хуй пососать
Аноним 24/05/19 Птн 01:55:39 48518435
>>485174
Если я правильно понял, сейбр в ближайшие полтора года будет проходить полноценные испытания вместе с прекулером, новость на сайте esa была, показывали строительство тестовой площадки
Аноним 24/05/19 Птн 05:00:33 48519436
А можно ли солярку в качестве ракетного топлива?
Что будет, что изменится?
А бензин?
А СПГ?
...
А МАЗУТ?
Аноним 24/05/19 Птн 07:29:57 48522637
>>485194
А можно колеса квадратные? А ромбические? А треугольные?... А можно из обедненного урана?
Аноним 24/05/19 Птн 07:32:55 48522738
tenor (2).gif (164Кб, 789x258)
789x258
>>485226
>А можно колеса квадратные?
Да, если ездить по специально спроектированной дороге рилейтед.
>А ромбические? А треугольные?
Абналогично, с поправкой на колею.
>А можно из обедненного урана?
Можно, но непрактично - уран хрупкий и тяжелый, при езде по кочкам будет трясти и колеса развалятся.

А теперь на мой ответ давай.
Аноним 24/05/19 Птн 07:53:21 48522839
>>485227
Можно, можно, можно, можно. Ракета полетит. Изменится конструкция двигателя и удельный импульс. Была серия в разрушителях мифов, где они в качестве топлива использовали какое-то совсем полное говно.
Аноним 24/05/19 Птн 07:54:07 48522940
>>485228
За ответ, конечно, спасибо, но он не настолько подробный, как хотелось бы.
Аноним 24/05/19 Птн 08:36:11 48523441
>>485194
Обычные соляра, бензин, тем более мазут - вряд ли, нужна особо высокая степень очистки для фракций, иначе получится неравномерное горение и неизбежный бабах. Ракетные движки гораздо более привередливы к чистоте, чем воздушно-реактивные, а те гораздо более чем поршневые.

СПГ то же самое, хотя его, по-моему, проще очистить и получить метан например.

Всячески рекомендую читнуть Ignition! Джона Кларка
https://archive.org/details/ignition_201612/page/n11
Там из первых уст описан процесс ебли с подбором и очисткой углеводородов для ракетных движков, в красках, фаерболах и языках пламени
Аноним 24/05/19 Птн 08:39:39 48523542
>>485234
>Ракетные движки гораздо более привередливы к чистоте, чем воздушно-реактивные, а те гораздо более чем поршневые.
А я чето слышал, что газотурбинные моторы могут хоть на угле (мелкодисперсном) работать, им насрать на топливо типа.

>Всячески рекомендую читнуть Ignition! Джона Кларка
>https://archive.org/details/ignition_201612/page/n11
Благодарю за качественное пополнение в мою читалку!
Аноним 24/05/19 Птн 09:34:49 48524043
>>485235
А какая связь газотурбин и ракетных двигателей, ты к чему это
>>485194
Любую горючую хуйню можно, особенно в гибридных ракетах, вопрос только в эффективности - почти все вещества намного хуже используемых, а те что лучше фтор, бораны - нецелесообразны
Аноним 24/05/19 Птн 09:59:20 48524644
>>485240
>А какая связь газотурбин и ракетных двигателей, ты к чему это
так это одно и тоже
Аноним 24/05/19 Птн 10:52:03 48526045
>>485246
Ну нет, лол. А вообще, привередливость к чистоте горючки в ракетных берется от мощности и экономии массы. Там гигаватты могут быть, а камера сгорания и сопло подобраны в тютельку. Маленький чих у этой прорвы может оказаться такой мощности, что распидорасит все.
Аноним 24/05/19 Птн 10:54:57 48526246
>>485260
И да, особенно важен процесс запуска - при неравномерном распространении может получиться хардстарт, может так ебануть, что обломков не соберешь. Вот в Ignition!, приведенной выше, результат хардстарта буквально на первой-второй странице. и такая KSP там всю книгу
Аноним 24/05/19 Птн 11:04:59 48526647
155453644215783[...].png (164Кб, 400x357)
400x357
>>485240
>Любую горючую хуйню можно, особенно в гибридных ракетах
Все опять же зависит от масштабов. Если любительская пукалка в тисках в гараже может завестись на дровах и чудом не ебануть, то практический движок обычно пидорасит нахер. Вон даже на мелком казалось бы гибриднике SS2 два или три раза топливо меняли из-за неравномерности горения. А все потому, что из-за экономии массы все подогнано в край.
Аноним 24/05/19 Птн 14:42:18 48532048
>>485266
У тебя причинная связь хромает
Аноним 24/05/19 Птн 16:16:33 48535149
Будет ли у гаусс-винтовки отдача? Пропеллента тонет.
Аноним 24/05/19 Птн 16:17:31 48535250
Аноним 24/05/19 Птн 16:25:35 48535551
>>485351
Будет.
Третий заон Ньютона.
Как пуля отталкивается от ствола, чтобы ускориться, так и ствол от пули.
Аноним 24/05/19 Птн 16:28:31 48535652
>>485352
Я знаю про третий закон и закон сохрания импульса, но не могу правильно его сюда применить. В обычном огнестреле порох по сути делает из оружия ракету, толкая всю конструецию назад плюс затвор отъезжает назад и бьет по раме. А в гавусе что будет толкать конструкцию назад? Единственное что лезет в голову - это катушки будут "отталкиваться" от пули через магнитное поле.
Аноним 24/05/19 Птн 16:35:40 48535753
>>485356
а что такое ракета? это выброс массы в одну сторону и движение в другую. пуля имеет массу, значит есть выброс массы и движение в другую сторону.
Аноним 24/05/19 Птн 16:40:42 48535954
>>485356
>это катушки будут "отталкиваться" от пули через магнитное поле
Именно так и будет.
Аноним 24/05/19 Птн 17:28:41 48536655
Аноним 24/05/19 Птн 17:41:54 48536856
>>485366
С размером связи крайне мало, двигатели делают на тщательно выверенных компонентах потому они должны работать максимально эффективно, а не потому что они большие
Ничто не мешает сделать огромных движок на говне, вопрос только в том нахуй он нужен если на нормальных компонентах тяга и импульс будут в разы выше
Аноним 24/05/19 Птн 18:14:10 48538557
methalox hardst[...].mp4 (1306Кб, 426x190, 00:00:33)
426x190
>>485368
Ты похоже не умеешь воспринимать написанное. Я не про размер говорю, а про снижение массы. Ясен хуй можно сделать забетонированный демонстратор с ебически толстыми стенками, чтоб выдержал неравномерное сгорание и не взорвался на старте, но так не делают, потому что никуда он не полетит - слишком тяжелый.

>Ничто не мешает сделать огромных движок на говне
Мешает дохуя чего. Далеко не любые (даже энергетически выгодные) компоненты дают ровную реакцию, даже если они абсолютно чисты и условия идеальные. Реальный движок крайне хрупок по сравнению с прорвой энергии, которую он извлекает и направляет в нужную сторону. Небольшое отклонение от накатанной, в результате неправильно рассчитанной динамики, или неподходящих примесей, и пизда. Шебмрилейтед например вместо стандартной тонны тяги получил резкий удар в три тонны, как думаешь, на пользу это ему пошло? Это малюсенький хардстарт, на заре проектирования ЖРД, когда ошибки были в порядке вещей, это чаще выглядело как распидорас всего стенда до основания.

Я не зря Ignition! выше советовал, там от первого лица автор повествует про несколько десятилетий подбора компонент топлива, про скорость сгорания, про стабильность реакции, про постоянные бабахи которые происходили из-за этой хуйни и уносили с собой создателей, про то как создавались марки реактивных углеводородных топлив и очистку фракций, ибо ни бензин, ни обычный керосин, ни дизтопливо не подходили. Компоненты есть и получше по энергетике, но остановились на применяемых по разным другим соображениям, включая скорость реакции и ровность горения (а не только удобство обращения с ними, плотность или хранимость).
Аноним 24/05/19 Птн 18:15:55 48538658
>>485385
Я знал что на этом видео будет с первых секунд. Очень классное показательное видео.
Есть моар подобного, наглядного?
Аноним 24/05/19 Птн 18:38:31 48538959
изображение.png (26Кб, 613x535)
613x535
Господа, я дико извиняюсь, но что за хуйня?
Ебашу значит в orbiter 2016, втыкаю в transx, пытаюсь слетать к марсу. После отлета от земли, в тот момент когда удаляется первый стейдж, transx перестает рисовать какую либо графику вообще. Пробовал перезапускать игру, не помогло.
ЧЯДНТ?
Аноним 24/05/19 Птн 18:43:18 48539160
>>485389
У нас же где-то был Орбитер-тред? В каталоге не найти почему-то.
Аноним 24/05/19 Птн 18:46:21 48539461
111112.png (915Кб, 1221x688)
1221x688
Подскажите это блик или созвездие какое, али еще что-то?
Аноним 24/05/19 Птн 18:47:34 48539562
>>485389 >>485391
Был, но настолько мертвый что потонул даже тут.
Банальная наводка - а у тебя версии-то совпадают? транс-экса и орбитера.
24/05/19 Птн 18:51:54 48539763
>>485394
1. если в высокой широте типа авроры может быть?
2. может что на камере блик какой
3. пыль от станции может слетела, какой-нибудь двигатель включался?
Аноним 24/05/19 Птн 18:52:59 48539864
>>485394
Блик.
Созвездия быть не могут видны в принципе, земля же в кадре.
Это как в светлой комнате снимать в сторону ночного окна и пытаться что-то там разглядеть. Открытого окна, чтобы не бликовало.
Яркости несопоставимы, камеры такое не берут.
Аноним 24/05/19 Птн 18:55:04 48539965
S036221770.jpg (1199Кб, 4256x2832)
4256x2832
>>485394
Это малафья на одной из камер HDEV, там гермоконтейнер с термостатом и обычная коммерческая камера.
Аноним 24/05/19 Птн 18:59:10 48540066
А почему до сих пор можно найти камни из катархея если есть движение плит и они по идее должны были кучу раз друг под друга залазить и переварить поверхность планеты по многу раз?
Аноним 24/05/19 Птн 19:03:43 48540267
>>485385
Еще раз.
Моя позиция в том что требования к компонентам исходят исключительно из, скажем так, серьезности и функциональности ракеты, а не размеров.
Аноним 24/05/19 Птн 19:17:57 48540968
>>485395
Бля, по ходу ты прав. У меня вот этот:
TransX V3.14 for orbiter 2010
Где отосрать для 2016?
Аноним 24/05/19 Птн 19:18:23 48541069
>>485402
В таком случае ты отвечаешь на что угодно, но не на мой пост. Я всего лишь ответил на вопрос "что будет если бензин в ракетный движок залить". Скорее всего он просто ебанёт, т.к. состав автобензинов слишком плавает. Т-1, РГ-1, RP-1 специально были созданы как особо чистые авиаварианты керосина, а не потому что энергетически выгодней.

>требования к компонентам исходят исключительно из, скажем так, серьезности и функциональности ракеты
И это кстати тоже хуйня, там требований миллион.
Аноним 24/05/19 Птн 19:19:13 48541270
>>485410
А нельзя сделать чисты бензин?
Чистую солярку?
...
Чистый МАЗУТ?
Аноним 24/05/19 Птн 19:21:23 48541471
>>485412
Можно, получится авиамазут. нахуй никому не нужный
Аноним 24/05/19 Птн 19:22:04 48541572
>>485414
Проиграл с идеи авиамазута и ракеты на мазуте.
До сих пор хихикаю как даун.
Аноним 24/05/19 Птн 19:22:54 48541673
>>485409
Всё, отбой, нагуглил. Щас попробуемс.
Аноним 24/05/19 Птн 19:23:21 48541774
>>485409
В ангаре, конечно, где ж еще.
https://www.orbithangar.com/searchid.php?ID=6393

Не забудь еще соответствующую тему форума глянуть, в теле поста ссылка и внизу юзер комментс ссылка.
Аноним 24/05/19 Птн 19:52:58 48542575
>>485410
>В таком случае ты отвечаешь на что угодно, но не на мой пост
бля ну нить не теряй
>Любую горючую хуйню можно, особенно в гибридных ракетах
>Все опять же зависит от масштабов
>Я не про размер говорю, а про снижение массы. Ясен хуй можно сделать забетонированный демонстратор с ебически толстыми стенками, чтоб выдержал неравномерное сгорание и не взорвался на старте, но так не делают, потому что никуда он не полетит - слишком тяжелый.
снижение массы и прочие параметры - это следствие назначения двигателей
>Я всего лишь ответил на вопрос "что будет если бензин в ракетный движок залить". Скорее всего он просто ебанёт, т.к. состав автобензинов слишком плавает
ебанет от бензина он потому что вследствии своего назначения двигатели должны быть максимально эффективными, следовательно они должны работать на предельных параметрах. с размером и масштабами, как ты говорил, связи нет - никто же не делает мелкие орбитальные пукалки и рсу на говне,
короче хуй знает зачем ты вообще поднял тему размеров, правильный ответ должен быть таким : "создать реактивный двигатель можно на любом горючем веществе, создать эффективно работающий и безопасный двигатель можно только на некоторых чистых веществах водород-кислород или тщательно подобранных компонентах с определенным составом керосин, гидразины, петардные смеси и прочее"
Аноним 24/05/19 Птн 19:59:39 48542776
>>485425
>короче хуй знает зачем ты вообще поднял тему размеров
Не, у тебя явно проблемы с пониманием. Я её не поднимал. Я наоборот сказал, что ничего от них не зависит, см.>>485266
>даже на мелком казалось бы гибриднике
А масштабы это вообще не про размер было, а масштабы затеи.

Короче проехали, ты прицепился непонятно к чему вообще, и какую-то свою хрень гнешь
Аноним 24/05/19 Птн 20:02:23 48542877
>>485400
Постоянно возникает и переваривается только океаническая кора, сами континентальные плиты гораздо стабильнее.
Аноним 24/05/19 Птн 20:04:01 48542978
>>485428
Воу, спасибо.
А почему так?
Где можно в доступном для олигофренов сжатом виде ознакомиться с этим?
Желательно в виде научпопа на английском.
Аноним 24/05/19 Птн 20:04:41 48543079
>>485425
>создать реактивный двигатель можно на любом горючем веществе
Нет нельзя, блять!
На любом говне можно демонстратор создать, зажатый в тисках и нелетающий. Которых на ютубе у кузьмичей полно. Или ракетные санки, в которых более-менее похуй на массу, или что-то в этом духе. Чтобы ЖРД двигал ракету, даже любительскую хуевину на километр поднять, надо чтобы он легкий был и хрупкий, относительно управляемых энергий. Поэтому, кстати, почти и нет кустарных летающих ЖРД - они либо требуют довольно точной машинерии, либо перетяжелены.
Аноним 24/05/19 Птн 20:06:18 48543180
>>485430
Самая жопа же в хрупкости сопла?
Почему бы не обойтись тогда без сопла лаваля? ТНА похуй на вид топлива по идее.
Аноним 24/05/19 Птн 20:07:45 48543381
>>485427
>масштабы затеи
бля сразу бы сказал
>>485430
я не понял, почему у тебя нелетающий двигатель не двигатель?
Аноним 24/05/19 Птн 20:10:55 48543582
>>485433
>я не понял, почему у тебя нелетающий двигатель не двигатель?
Потому что так можно договориться до того, что ацетиленовый сварочник - двигатель, и фонарик тоже. фотонный
Аноним 24/05/19 Птн 20:12:51 48543683
Аноним 24/05/19 Птн 20:22:13 48544084
>>485435
Ну, вопрос в применении и идеи во время создания
Если я соберу хуйту которая сможет волочится по земле на своей тяге это же будет двигатель, не так ли?
Аноним 25/05/19 Суб 09:15:18 48555085
Аноним 25/05/19 Суб 11:40:09 48556586
Почему масконы на луне настолько сильно на орбиту влияют?
Если такая большая проблема, почему нельзя просто орбиту выше сделать?
Аноним 25/05/19 Суб 12:08:18 48556787
>>485565
масконы страшны если ты о них ничего не знаешь и выбрал орбиту наобум очень низкую. Если ты о них знаешь то мы можешь выбрать орбиту либо более высокую, либо более эллиптическую, либо выбрать тщательно выверенную frozen orbit, либо заниматься поддержкой орбиты как на МКС, следя за высотой.
Аноним 25/05/19 Суб 19:20:40 48562488
>>485565
> Почему масконы на луне настолько сильно на орбиту влияют?
Осне большая разница в плотности. Есть места на краю масконов, где вертикаль отклонена на градусы.

>Если такая большая проблема, почему нельзя просто орбиту выше сделать?
Потому что часто нужна ниже. При любой посадке это тоже проблема.
Аноним 25/05/19 Суб 19:47:31 48563889
photo2019-05-15[...].jpg (126Кб, 706x1022)
706x1022
Поясните, что тут говорится?
https://zen.yandex.ru/media/funscience/merkurii-deistvitelno-mojet-okazatsia-samoi-blizkoi-k-zemle-planetoi-5c8f6b0de6d4d400b3addcc2
Я что-то так и не понял, почему из этой статьи выходит, что Меркурий - ближайшая к Земле планета. Какая-то подборка статистик. Я прям чувствую, что тут подвох, но в чём, никак в толк не возьму
Аноним 25/05/19 Суб 19:51:41 48563990
>>485638
Хули непонятно-то? Венера ближе всего _бывает_ к Земле, но в среднем дальше, чем Меркурий.
Аноним 25/05/19 Суб 19:53:35 48564191
>>485639
То есть, именно Меркурий нужно использовать для гравиманёвров хуиты с Земли?

Абажжи, ёбана. Но тогда получается, что и Земля к Сатурну будет ближе чем Юпитер.
Чё за бред.
Аноним 25/05/19 Суб 19:57:33 48564292
>>485641
>именно Меркурий нужно использовать для гравиманёвров хуиты с Земли
Схуяли? "Ближе" не значит, что к нему проще подлететь. Да и масса у него маленькая, а находится он глубже в гравиколодце Солнца.
> тогда получается
Не факт, подозреваю, что там эксцентриситет орбиты влияет. Орбиты планет ведь не совсем круглые.
Аноним 25/05/19 Суб 22:37:59 48565693
>>485641
> То есть, именно Меркурий нужно использовать для гравиманёвров хуиты с Земли?
С хуя ли? В чем логика такогт вывода?

> Абажжи, ёбана. Но тогда получается, что и Земля к Сатурну будет ближе чем Юпитер.
> Чё за бред.
Да, возможно, в среднем чаще к Сатурну ближе оказывается Земля, чем Юпитер.
Другое дело, что данный факт вообще ни на что не влияет. Дело в том, что в космосе все постоянно находится в движении, поэтому понятия "ближе", "дальше" в некоторых случаях не так важны, как "скорость", "масса", и т.д. И тем более когда речь идет о средних значениях. Среднее расстояние, блядь, да нахуй кому оно нужно
Аноним 26/05/19 Вск 00:06:32 48566994
Сап. Поясните за космическое производство: типа я с бухим батей спорил сегодня, зашла речь о том, что в Раше космос не мертв, и мы пендосам движки поставляем. Я смутно помню, что это типа мифа, но хотелось бы точных подтверждений (или опровержений)
Аноним 26/05/19 Вск 00:18:05 48567295
Аноним 26/05/19 Вск 00:26:17 48567396
>>485672
Окей. Так космос этой стране жив или как? На космических каналах в ютубе говорят, что отрасль проебали, но откуда движки тогда? Наследие совка?
Аноним 26/05/19 Вск 00:36:22 48567497
Каюсь, сразу не загуглил. Рд-180 в 90-е разработали, до 14 производили. Рд-181 не нашёл инфы на педивикии. Так ещё больше вопросов: поставки до сих пор происходят? Если да, то хуяли в 14 завершили производство. Сори за тупость: рубит уже пиздец
Аноним 26/05/19 Вск 01:59:45 48568698
>>485669 >>485673
Долго расписывать, это надо в роскосмос тред. Всё мертво, опыт давно потерян, т.к. разные области индустрии простаивают (в смысле вообще нихуя не делают) от 8 до 20 лет приблизительно, интереса к ним нет ни у государства, ни у бизнеса; в обозримом будущем (очень коротком, как обычно) тоже нихуя не предвидится. И дело не в деньгах. Исключения - ИСС и еще несколько контор, у которых всегда будет мелкий госзаказ на связь и ДЗЗ. У пилотируемой вообще 0 шансов после МКС. Но мозги нам ебать будут очень долго.
Аноним 26/05/19 Вск 02:03:42 48568799
>>485674
А мог бы зайти и на сайт производителя двигателей.
РД-181 производится с 2015. РД-180 в производстве тоже в настоящее время.
Аноним 26/05/19 Вск 08:18:39 485717100
Почему чёрные дуры сохраняют эл. заряд, хоть и недолго? А магнитное поле, например, нет.
Аноним 26/05/19 Вск 09:57:06 485737101
На орбите Земли вращается МИР (или что там сейчас вращается). Ему навстречу по этой же орбите/под углом, по прямой траектории столкновения к станции несётся небольшой астероид/метеор. Космическое тело врезается в космическую станцию. Допустим, станция сделана из суперкрутых материалов и не сломалась от такого столкновения, однако удар оказался таковым, что удар полностью нивелировал скоростью движения по орбите этой станции, сделав её нулевой относительно орбиты. Из-за этого закономерно это рухлядь начинает падать на Землю.
Собственно, вопрос: насколько быстро это груда металла упадёт с небесной тверди на земную?
Можно ли будет успеть просчитать траекторию падения?
А если \этот хлам будет падать на большой город, моно ли будет как-то спастись? Кроме бункеров и прочего ведь падение такой штуки должно мало того что сопровождать дождём из осколков, так ещё и кинетическая энергия в тротилловом эквиваленте (Мужик_с_ракеты_упал.пнг).
И ещё, если подумать логически, то помимо остановления вращения по орбите стнации, камушек-то тоже обязан остановиться, разве нет? Значит, и он падать начнёт. И в то же место, что и станция, разве нет?
То есть, гипотетический урон в подобном сценарии огромен
Аноним 26/05/19 Вск 11:22:13 485745102
large.jpg (21Кб, 494x484)
494x484
Аноним 26/05/19 Вск 12:04:42 485751103
>>485737
Вопрос сформулирован подегенератски, но попробую ответить. Предположим, что их нерасколошматило в осколки (что и произошло бы), предположем каким то хуем они остановились. То есть, что, если МКС (не МИР, сука) замедлит орбитальную скорость до нуля? Высота МКС 408 км. Предположим, что воздуха нихуя нет. Тогда у нас есть волшебная формула t = корень(2h/g), где g — ускорение саободного падения. t = корень(2 408000 / 9.8) ~= 290 с. v = g t ~= 2.8 км/сек. То есть скорость приземления будет всего нихуя (как у истребителя, уебавшегося носом на полной скорости в землю. Если добавить воздух и предположить, что станция каким то хуем не развалится в воздухе, то сопротивление будет сильно заглушать ускорение и приземление при ударе максимум провалит крышу какого-нибудь дома (если очень повезёт попасть).
Короче, метеориты гораздо опаснее, потому что они уже из космоса летят в землю на неебической скорости от 11 до 72 км/сек, а вышеописанное ничего, кроме пука в лужу не сделает.
Аноним 26/05/19 Вск 12:06:24 485752104
>>485751
предположим
2 x 408000


быстрофикс
Аноним 26/05/19 Вск 12:07:28 485753105
даблфейспалм1.jpg (44Кб, 600x530)
600x530
>>485737
пиздец ты тупой, даже по шкале градации тупости не найти насколько ты тупой
Аноним 26/05/19 Вск 12:16:09 485756106
>>485753
Добро пожаловать в реальный мир, анон. 99% людей вообще не ебут как работают космос и физика, но анону хватило смелости не промолчать и раз уж мы в ТТВ, то нужно быть готовым отвечать на любые вопросы. Лучше задавать тупейшие вопросы, вместо того, чтобы держать их в себе и не развиваться никак.
Аноним 26/05/19 Вск 12:52:19 485767107
>>485751
Хорошо, а если два липких тела (две кометы из клея "наномомент") врежутся в друг друга одинаковой скорости так, на противоположеной по вектору, упадёт ли их совместная пост-ударная скорость настолько, что они упадут на Землю?
За сколько они упадут?
Насколько разрушительной будет кинетическая энергия от падения?
Аноним 26/05/19 Вск 13:01:51 485768108
>>485767
Все выше применимо и здесь. Будет пук в лужу.
Аноним 26/05/19 Вск 13:41:24 485771109
Non Cerebri - N[...].png (37Кб, 640x480)
640x480
>>485756
Раз такая пляска, может объясните мне, что такого взрывного в том посте?
Аноним 26/05/19 Вск 13:59:05 485786110
>>485751
Сколько там весит мкс? 200 тонн? Кинетическая энергия=200000 2800 2800 / 2 = 7.84 10^11 Дж
Метеорит массой 1 тонна и скорость 30 км/с = 1000
30000 30000 / 2 = 4.5 10^11 дж
Аноним 26/05/19 Вск 14:12:16 485792111
1506913491056.png (97Кб, 397x189)
397x189
>>485737
Что за хуйню я читаю? Пиздец, какой-то наркоманский бред.

Но ладно, если свести твой вопрос к "что будет, если на орбите лоб в лоб столкнутся два одинаковых идеальных объекта с одинаковой скоростью", то
>насколько быстро
Зависит от высоты, а ускорение свободного падения известно, задача для 5 класса средней школы. Атмосфера затормозит прилично, немного отклонит друг от друга. С низкой орбиты - несколько минут, результирующая скорость будет не очень большая.
Аноним 26/05/19 Вск 14:42:39 485794112
>>485792
А спрогнозировать место падения можно будет успеть до падения?
Ведь ещё и планета крутится и от удара траектория может сместиться
Аноним 26/05/19 Вск 14:45:14 485796113
>>484908 (OP)
>Можно упасть в пузырь альбукерке, наса уже почти надула его.

Может ли расширение пространства толкать, а точнее создавать силу, что будет толкать тело - упорядоченную группу атомов, а не прост расширять пространство и расстояние между атомами?
Аноним 26/05/19 Вск 14:45:43 485797114
>>485794
>А спрогнозировать место падения можно будет успеть до падения?
Да
Аноним 26/05/19 Вск 15:10:35 485812115
>>485786
Окей, давай добавим сопротивление и посчитаем терминальную скорость. Выйдет не больше ста метров в секунду.
Аноним 26/05/19 Вск 15:12:21 485813116
>>485812
Ну не сто. Зависит от объекта. Астероид побольше, чудом не разрушившаяся бочка или обломки - поменьше
мимо
Аноним 27/05/19 Пнд 06:11:47 485938117
>>485786
МКС весит 400 ~тонн
Челябинский метеорит весил больше 9000 тонн.
Аноним 27/05/19 Пнд 10:44:42 485952118
pic4d4e5a844d74[...].jpg (55Кб, 420x280)
420x280
Аноним 27/05/19 Пнд 11:19:31 485953119
Аноним 27/05/19 Пнд 13:16:27 485967120
>>485771
Во-первых, пост уже начинается весело. Я не знаю в какой жопе мира надо жить, чтобы не знать что сейчас на орбите МКС, а Мир давно почил в глубине Тихого океана.

Во-вторых
>Ему навстречу по этой же орбите/под углом, по прямой траектории
Таки по этой орбите, под углом, или по прямой? Из контекста становится понятно, что автор имел ввиду, что вектор скорости астероида должен быть направлен в противоположную сторону в момент столкновения.

3. Зачем вообще приплетать сюда станцию, если она не разрушается? Автор мог бы написать "два тела движутся навстречу друг другу". А, но станция, по словам автора вопроса, разрушается в атмосфере, поэтому нужно спасаться от дождя из осколков.

4. Первая космическая = 7.9 км/с. МКС движется чуть медленнее. Если она полностью остановится, то ее скорость будет равна 0. Дальше - задача из школьного учебника.

5. В общем, переформулирую вопрос автора, как его бы задал нормальный человек:

Что будет, если в МКС врежется другое тело таким образом, что скорость обоих тел упадет до нуля, но станция не разрушится. Каковы последствия падения этих двух тел на Землю?

Я не часто здесь пощу, только бы не проебаться, я не хочу чтобы меня стебали в ТТВ
Аноним 27/05/19 Пнд 17:35:18 486013121
1558967701111.jpg (47Кб, 575x385)
575x385
Как сделали это фото?
Аноним 27/05/19 Пнд 17:40:53 486015122
>>486013
при помощи камеры
Аноним 27/05/19 Пнд 17:50:03 486018123
>>486015
А камера где располагалась?
инб4 в космосе
На каком космическом аппарате была эта камера?
Аноним 27/05/19 Пнд 18:02:46 486019124
>>486018
Я ебу? Я тебе должен расследование проводить на тему того какие аппараты это, в каком году, где ты взял это фото? А ты хули делать будешь?
Аноним 27/05/19 Пнд 18:04:59 486020125
>>486019
Хорошие вопросы.
>Я ебу?
Судя по реакции - нет.
>Я тебе должен расследование проводить на тему того какие аппараты это, в каком году, где ты взял это фото?
Было бы неплохо, если бы провел. Орбитальных станций не так и много.
>А ты хули делать будешь?
Мастурбировать и играть в игры.

Спасибо за вопросы, задавай еще.
Аноним 27/05/19 Пнд 18:17:38 486022126
>>486019
>>486015
Нахуй ты высираешься здесь если ты даже не в курсе когда была сделана эта фотография ебанько?
И вообще это не совсем фотография, а кадр видеозаписи.
Аноним 27/05/19 Пнд 18:18:37 486023127
Такие же есть и с мкс. Только как их делают? Через телескоп?
Аноним 27/05/19 Пнд 18:30:08 486028128
>>486023
К космическим станциям обычно летают что? Правильно, космические же корабли.
Если то был кадр с видеозаписи, то мог быть Прогресс. Или огурцы из Союза сфоткали. Что, обычно, и делается.
Аноним 27/05/19 Пнд 19:41:01 486034129
>>486022
>если ты даже не в курсе когда была сделана эта фотография
То есть вопрос был адресован тому, кто знает наизусть видео всех космических станций и может опознать источник видео всего лишь по одному кадру?
Аноним 28/05/19 Втр 03:02:14 486101130
>>484908 (OP)
Как так получилось, что более чем полвека назад люди на Луну летали, а сейчас проблема отправить человека на орбиту Земли?
Аноним 28/05/19 Втр 06:44:23 486110131
Аноним 28/05/19 Втр 07:33:02 486112132
>>486101
Где проблема?
Щас аппараты к марсу летают,к сатурну, мягко садятся на астероиды и кометы
Щас охуеть как продвинулась космонавтика, а человек в космосе просто юзлес, вот и не пускают
Хотя на мкс часто летают люди, а на луну в 2024 и так наса отправляют людей
Аноним 28/05/19 Втр 09:04:46 486124133
>>486101
Так на луну для флаговтыка и чтобы ДАТЬ ПОСОСАТЬ летали, тратя миллиарды нефти. А сейчас уже не нужно выебываться перед другой страной.
Аноним 28/05/19 Втр 09:44:37 486131134
>>486110
>Что думаете?
Не стоит в спейсач тащить статьи из непрофильной мурзилки, вот что я думаю. Ни ссылок на оригинальную работу, нихуя, один кликбейт и перевирание.

Но ладно, давай разберем, дело в том что озеро это нашли еще год назад по профилям радара MARSIS с аппарата Mars Express, датированным 2012-2015, вот статья
https://dx.doi.org/10.1126/science.aar7268 , пикчи в твоей мурзилке оттуда, смысл тоже. Может речь о том, что подтвердили другим методом или нашли другое озеро, но новых работ по этой теме я не вижу ни в Icarus, ни в других планетологических журналах, так я как профессиональный телепат мирового уровня склоняюсь к тому что они просто переврали прошлогоднюю статью и выдали за новую.

Вот поэтому не стоит тащить сюда всякое говно из невнятных источников, а то будем как мартышки обсуждать какую-то виртуальную хуйню без связи с тем что реально произошло.

Касаемо воды на Марсе - ну збс чо. Если подтвердят другим методом и к результатам будет больше доверия а может даже и так, это будет один из первейших кандидатов на исследование с поверхности, через АМС или даже космолюдьми, если таковые будут на Марсе когда-нибудь. В любом случае, какие-либо подробности или опровержения будут через годы.
Аноним 28/05/19 Втр 09:45:02 486132135
Аноним 28/05/19 Втр 12:13:40 486157136
>>486101
Нет проблемы. Нет бабок на это. Потому что нахуй никому не надо.
Аноним 28/05/19 Втр 12:35:48 486167137
>>486112
>Где проблема?
Американцы например уже сколько лет не летают. В рахе тоже никак новый корабль построить не могут. Раньше в 57-ом году первый спутник, в 61-ом первый человек в космосе, в 69-ом уже на Луне. А сейчас у Маска какая-то неполадка с кораблем и уже неизвестно на сколько лет пилотируемый запуск откладывается, не говоря уже о том сколько лет его до этого делали.
>>486124
>тратя миллиарды нефти.
Ну первый раз еще можно понять все затраты и трудности, но когда уже есть опыт, новые материалы, вычислительная техника и вообще прогресс во всех областях это как-то странно.
Аноним 28/05/19 Втр 12:55:27 486171138
>>486101
В том же, в чем и у остальных - застой в космической отрасли после спадения пропагандистской и милитари нагрузки времен холодной войны. Любая организация рано или поздно замедляется, любой процесс останавливается, нужны конкуренция и приход новых, чего почти нет. Пейсх слегка расшевелили болото, делая довольно банальные по меркам космических гонок вещи. Вторая причина - раньше это делали усилиями целых стран, сейчас никто таких денег тратить не будет, тем более что на какие-то принципиально новые вехи за сжатые сроки нужны принципиально новые суммы, по сравнению с которыми даже аполлон - возня в песочнице. Нужен нормальный бизнес, а пилотируемая космонавтика бизнесу не особо интересна.
Аноним 28/05/19 Втр 12:59:53 486172139
>>486167
>но когда уже есть опыт, новые материалы, вычислительная техника и вообще прогресс во всех областях это как-то странно
Весь этот прогресс не отменяет принципиальной сложности пилотируемой космонавтики (сложность не в смысле "трудно", а "заморочено"), а это и есть основной источник её стоимости.

Ну и risk aversion, да. Регулярная доставка огурцов в космос еще не перевалила за критическую частоту, а безопасность хотят уже сейчас. В результате не получают ни того, ни другого, получая уловку-22.
Аноним 28/05/19 Втр 15:12:32 486192140
>>486167
Посмотри сколько бабла в космос вливалось тогда и сейчас.
Вот и всё. Одна единственная причина.
Аноним 28/05/19 Втр 17:09:50 486222141
>>486167
>Раньше в 57-ом году первый спутник, в 61-ом первый человек в космосе, в 69-ом уже на Луне
Ну так между первыми европейцами в Америке и первыми европейскими колониями также широкий исторический перерыв присутствует.
Аноним 28/05/19 Втр 20:30:02 486279142
Как на Марсе с аккустикой? Если я буду на Марсе, сниму шлем и попытаюсь докричаться до другого человека, который снял шлем, но стоит на расстоянии в сто метров, то он меня не услшит, как бы сильно я не кричал?
Аноним 28/05/19 Втр 20:37:34 486280143
>>486279
>с аккустикой
С этим даже на Земле хуево.
Аноним 28/05/19 Втр 20:43:37 486281144
Аноним 28/05/19 Втр 21:08:59 486288145
С чем связано увеличение скорости этой борды? Некоторые треды висели годами, но теперь, по крайней мере 4 треда уже давно номерные. Скорость борды с каждой неделей растёт, хотя космических запусков не особо больше становится.
Аноним 28/05/19 Втр 21:12:12 486289146
>>486288
Кербанов вернули в раздел
Аноним 28/05/19 Втр 21:18:11 486291147
>>486288
Модеры перестали банить за каждый пук не по правилам.
Аноним 28/05/19 Втр 21:30:29 486298148
>>486288
Нет никакого увеличения. Просто шитпостинг сконцентрировался в паре тредов, остальное никому не интересно, как это было до превращения доски в /б/ во времена Розетты
Аноним 28/05/19 Втр 21:34:19 486300149
>>486291
Единственная адекватная инициатива. А то как в гулаге, по 2 поста в неделю с этими ебанутыми банами ни за что, номерными тредами и вонючими жуками.
Аноним 28/05/19 Втр 22:00:43 486304150
>>486300
Не, ну про жуков ты зря. Это ж икона доски!
Аноним 28/05/19 Втр 22:15:32 486308151
>>486300
Лучше 2 поста про космонавтику, чем 200 про рогозина и грымзу
Аноним 28/05/19 Втр 22:18:20 486309152
>>486308
Насколько я помню, во времена адовой анальной модерации 2 поста в неделю так же были про рогозина и грымзу.
Аноним 29/05/19 Срд 08:00:40 486345153
Как психологически готовят космонавтов?
Какое направление предпочтительнее?
Аноним 29/05/19 Срд 08:26:08 486346154
И ещё, какие именно лекарства есть в космосе на текущий момент ?
Из списка ноотропов, стимуляторов, транков, витаминов и т.п
И ещё, я слышал раньше и сейчас в космос берут такой препарат как фенибут, хотя он не доказал какой либо эффективность в плацебо контроллируемых тестах. Как так? Или такое возможно только на российской стороне?
Какая аптечка в сша?
Аноним 29/05/19 Срд 09:50:52 486365155
>>486300
>вонючими жуками
Пиздец, старейший тред на доске а может и на всем мейлаче, а он нос воротит.
Аноним 29/05/19 Срд 10:13:52 486369156
Аноним 29/05/19 Срд 10:27:04 486370157
>>486345
> Как психологически готовят космонавтов?
- во-первых, отсеивают психов из кандидатов
- дроч ситуаций до автоматизма и хладнокровия - в имитаторе капсулы с пожаром на борту, в бассейне для ВКД и т.д. и т.п.
- вместе живут в бочке некоторое время, чтоб в реальном полете не перегрызлись (что все равно случается иногда)
- общее обучение принятию решений в условиях непоняток, ограничений, дезориентации
и естественно непрерывное наблюдение и снятие метрик

>И ещё, я слышал раньше и сейчас в космос берут такой препарат как фенибут, хотя он не доказал какой либо эффективность в плацебо контроллируемых тестах. Как так? Или такое возможно только на российской стороне?
Не в курсе, но не удивлюсь если это так, с этим всегда были и есть проблемы.
Аноним 29/05/19 Срд 11:09:13 486385158
Аноним 29/05/19 Срд 11:13:10 486386159
>>486370
> ну тип готовят хуё моё в бочке сидят учатся там тип хуй знает))))
Я вроде конкретно спрашивал, какой подход исповедуют в наса
Аноним 29/05/19 Срд 12:16:31 486408160
>>486385
Ну тогда на этой доске как минимум.
Аноним 29/05/19 Срд 17:23:34 486478161
Что подразумевал Кеннеди под "do the other things"?
29/05/19 Срд 17:29:05 486482162
>>486478
>We choose to go to the moon in this decade and do the other things, not because they are easy, but because they are hard
Науку на Луне, космические станции, межпланетные зонды? Смотри в контексте того времени.
Аноним 29/05/19 Срд 20:00:47 486520163
Как масса влияет на пространство?
Аноним 29/05/19 Срд 20:16:47 486526164
Аноним 29/05/19 Срд 20:19:01 486528165
>>486520
Тебя это ебать не должно
Аноним 29/05/19 Срд 20:26:56 486530166
>>486291
Раньше моча лютовала дико. Я как-то словил бан за невинный вопрос, о том что будет, если космонавт, пролетая над горизонтом событий, сунет хуй за горизонт.
Аноним 29/05/19 Срд 20:31:29 486531167
>>486530
Может бан и был ответом на твой вопрос, глупенький.
Аноним 29/05/19 Срд 20:38:26 486532168
>>486530
>Раньше моча
Все логично, нет космоса - нет мочи.
Даже лахта забила на мертвый тред мертвой рогозы - поэтому моча уже не нужна.
Аноним 29/05/19 Срд 20:45:09 486533169
Аноним 29/05/19 Срд 20:58:44 486535170
>>486532
Иногда даже интересно, что за хуйня происходит в голове у людей, которые настолько зациклены на порашной повестке. Так и представляю, как такие утром случайно ударившись пальцем о быльце кровати или пролив на себя кофе кричат на всю квартиру АХ ТЫ ЁБАНЫЙ ПРОКЛЯТЫЙ РОГОЗИН/МАСК/ПОЛИТИЧЕСКИЙ ДЕЯТЕЛЬ США ИЛИ РОССИИ! и идут писать на спейсач про лахту или русофобов.
Аноним 29/05/19 Срд 21:25:46 486539171
>>486535
Лахта на месте, я спокоен.
Аноним 29/05/19 Срд 23:51:10 486546172
Аноним 30/05/19 Чтв 03:03:28 486568173
Почему интерферометрию делать в видимом спектре тяжелее чем в микроволнах?
Я хочу задействовать Кек и Очень Большой, например.
Почему нельзя?
Аноним 30/05/19 Чтв 04:13:43 486571174
>>486346
Спроси Артемьева, он вроде в соцсетях более-менее активный до сих пор.
Аноним 30/05/19 Чтв 07:47:51 486576175
>>486386
>Я вроде конкретно спрашивал, какой подход исповедуют в наса
Вроде? А может не вроде?
>>486345
>Как психологически готовят космонавтов?
>Какое направление предпочтительнее?

Энивей, в наса такой же подход. Какого конкретно ответа ты хотел?
Аноним 30/05/19 Чтв 07:55:24 486577176
>>486568
Потому что длина волны меньше. Отсюда резко возрастают требования к определению расстояний между приёмниками, к джиттеру линии связи и точному определению задержек, к вычислительным мощностям сведения всей этой хуйни в картинку.
Аноним 30/05/19 Чтв 08:00:37 486579177
>>486386
Нихуя ньюфаги охуевшие пошли.
>тупой вопрос хуйпойми о чем
>>обзорный ответ
>ррряяяя недостаточно подробно
>и воще я не то спрашевал
Аноним 30/05/19 Чтв 08:05:38 486581178
Почему не сделают хотя бы маленькую пусковую петлю 50-100км примерно, чтоб с ускорением 100-300жэ ништяки забрасывать? Митолы, воду. Построить робозавод, клепать звездолёты прямо там.
Аноним 30/05/19 Чтв 16:50:50 486657179
Аноним 30/05/19 Чтв 16:59:17 486659180
>>486581
Пора уже в шапку добавлять.
Аноним 30/05/19 Чтв 17:13:38 486660181
Почему когда Гагарин летал ему не дали кинокамеру или фотоаппарат хотя бы? Ведь эпохальное событие же.
Аноним 30/05/19 Чтв 18:17:58 486669182
2e23a4fe30292d7[...].jpg (25Кб, 334x300)
334x300
i0518rp.jpg (5Кб, 250x167)
250x167
>>486660
Ну почему, там была SSTV-телекамера К-100, снимавшая его, она же и на пленку фиксировала. Но качество было пиздос. Вообще, у совков всегда было туго с фото-видео-регистрацией. Дело даже не в элементной базе и не в секретности, просто сама идея что-то снимать на борту/с борта, а потом показывать - была непонятна. Даже эту камеру Гагарину запихнули не чтобы запечатлеть эпохальное событие, а чтобы следить за его состоянием. Из-за этого неприятия, и из-за пренебрежения вояками и индустрией аэрофотосъемкой наработки 40-х по оптике (которые были неплохи на то время + захваченные в Германии технологии) не особо развивались, в результате чего всю космическую эру совок сосал по фото, кино и видео аппаратуре, да и потом. А аналога Швеции, в которой Хассельблад делал лунные фотики, в варшавском блоке не было, и пленка хуевая была. Всё ДЗЗ опять же использовало сильно перетяжеленную оптику из-за этого. Даже блять захваченную в середине 50-х спецпленку с одного из американских шпионских стратостатов Genetrix использовали потом в станции Луна-3, ибо в совке такой не делали.
Аноним 30/05/19 Чтв 18:19:54 486670183
tv1.jpg (47Кб, 730x420)
730x420
>>486660
Во-первых, какие-то телевизионные камеры внутри были, которые самого Гагарина снимали.

Во-вторых, а нахуя ему камеру было давать? Что бы он с ней делал, свои ноги снимал? Гагарин мало того что весь полет лежал пристегнутым в кресле, так еще и катапультировался из капсулы Востока и приземлился на парашюте, с собой он ничего из капсулы забрать не мог, все об землю ебнулось.
Аноним 30/05/19 Чтв 18:29:12 486671184
>>486670
>Что бы он с ней делал, свои ноги снимал?
Мог бы Землю в илюминатор снимать.
Аноним 30/05/19 Чтв 20:31:14 486705185
12252532652.jpg (148Кб, 640x630)
640x630
fc61586e13b1842[...].jpg (145Кб, 381x500)
381x500
>>486670
Технически-то вполне решаемо, хоть и заморочки были бы. Например можно было бы на кресло захуярить фотоаппарат, а в кабину поставить зеркало. Чтобы не нужно было большое@тяжелое зеркало - сделать его чуть выпуклым, чтобы как на ширик получалось, а объектив длиннофокусный. Другое дело, что внутренности передового КА в газете показывать - это же гроб гроб режим кладбище враг подсмотрит, а обложка Науки и Жизни №5/1961 обошлась бы и иллюстрацией, по их мнению, чай не Лайф какой-нибудь загнивающий и не буржуазное насо, которое изначально придерживалось почти полной публичности.
Аноним 30/05/19 Чтв 20:39:37 486709186
>>486670
Камеру с собой в корабль, широкоугольный объектив, можно крючок и спусковой тросик. Пленку разрядить и в карман. Меркури вообще надевали на себя, но малый объем не стал помехой для селфи космочеловека в летающем гробу.
Аноним 31/05/19 Птн 00:15:24 486778187
>>486669
>Даже блять захваченную в середине 50-х спецпленку с одного из американских шпионских стратостатов Genetrix использовали потом в станции Луна-3, ибо в совке такой не делали.
Вот все про это говорят как про очевидный факт, но я ни разу пруфсов не видел, зато видел отсутствие упоминания этого у Чертока когда он про Луны писал.
Аноним 31/05/19 Птн 00:56:59 486789188
>>486778
Оригинальный источник этой кулстори - журнал НК за август 2000, инфа из первых рук кого-то из дидов.
>зато видел отсутствие упоминания этого у Чертока когда он про Луны писал
Да у Чертока дохрена чего нет, это ж не энциклопедия. Каманина того же почитать, там тоже то чего у него нет. А вообще если ты хочешь прямо историчности - мемуары самый ненадежный источник, и в исторической науке всерьез их никто не воспринимает. Так что дохуя баек, и эта, и чертоковские, и каманинские, могут оказаться пиздежом или приукрашенными, или перевранными, в конце концов. Имеем то что имеем.
Аноним 31/05/19 Птн 10:21:13 486830189
>>486669
>Даже блять захваченную в середине 50-х спецпленку с одного из американских шпионских стратостатов Genetrix использовали потом в станции Луна-3, ибо в совке такой не делали.
зачем использовать пленку если она все равно не вернется на землю и фоток с нее не сделать?
Аноним 31/05/19 Птн 10:26:48 486832190
>>486830
>зачем использовать пленку если она все равно не вернется на землю и фоток с нее не сделать?
Пленку потом медленно сканировали используя достаточно мощную лампочку и это уже отсылали.
Аноним 31/05/19 Птн 12:14:34 486848191
Горячо ли в центре Плутона и Тритона? Хотя там может быть горячо только из-за распада радиоэлементов, есть ли там горячая мантия ещё?
Аноним 31/05/19 Птн 12:32:05 486852192
Почему большие обсерватории делают один большой телескоп вместо сотни маленьких? Зеркалами соединить изображение со всех телескопов в одну точку и готово, или это хитрый план по попилу бабла?
Аноним 31/05/19 Птн 12:53:56 486854193
eelttelescopepi[...].jpg (282Кб, 1366x657)
1366x657
>>486852
>Зеркалами соединить изображение со всех телескопов в одну точку и готово
Ты не поверишь!
Аноним 31/05/19 Птн 13:00:33 486856194
Аноним 31/05/19 Птн 17:16:07 486889195
(Изображение JP[...].jpg (147Кб, 679x1024)
679x1024
>>484908 (OP)
прошу рассказать плюсы и минусы этого бинокля
Сам не разбираюсь в физике и астрономических наблюдениях, это первая покупка. Читал, что линзы портятся со временем, по краям плохо всё итд.
Аноним 31/05/19 Птн 17:16:33 486890196
Аноним 31/05/19 Птн 21:00:10 486903197
>>486889
Это город с орбиты в бинокль?
Аноним 31/05/19 Птн 21:20:58 486904198
>>486852
Смотря что ты имеешь в виду. Если получить одну картинку с кучи зеркал - это и так делается в большинстве больших телескопов. И не только потому, что большие зеркала делать дорого и сложно доставлять, но и потому, что адаптивная оптика и подобные ухищрения по компенсации атмосферы этого требуют. Если же ты имеешь в виду сотни маленьких картинок, и затем простакать - то просто так это не получится, число картинок в стаке растет нелинейно, и нормальной чувствительности таким способом не получить, нужно именно большую апертуру.
Аноним 31/05/19 Птн 21:25:38 486905199
Аноним 01/06/19 Суб 12:22:07 486960200
Аноним 01/06/19 Суб 13:15:22 486962201
СхемаСпэйсшаттла.jpg (217Кб, 1200x960)
1200x960
Если грузовой бак шаттла заполнить максимально возможным количеством топлива, можно ли будет на нем слетать до Марса и назад, или даже дальше?
Проблемы радиации и жизнеобеспечения игнорируем.
Аноним 01/06/19 Суб 13:30:08 486964202
>>486962
Не. 27.5т ПН, 80т заправленный орбитер. Допустим повезет он две тонны огурцов и чипсов для них, и тонну будут весить емкости, получится 23.5 тонны вонючки для движков с УИ 316с. А получить надо как минимум 4.25км/сек только до низкой орбиты Марса, и это прям хорошие окна.
Аноним 01/06/19 Суб 13:31:57 486965203
>>486962
Да там дохуя реальности придется игнорировать, какое уж там ЖО. Этот пепелац не мог летать в принципе без затенения Землей, слишком жарился на солнце. Потому пуски были сильно ограничены. А уж тем более НОО никуда не мог из-за этого улететь.
Аноним 01/06/19 Суб 13:33:32 486967204
>>486965
>А уж тем более дальше НОО
фикус
Аноним 01/06/19 Суб 13:36:09 486968205
>>486964
А, например, до Луны и назад хватит?
Аноним 01/06/19 Суб 14:24:14 486973206
>>486968
А тебе обязательно без внешнего бака?
Если взять водород/кислород то до марса нужно взять внешний бак размером примерно(!) 1/5 от шаттловского , плюс внутри.

Так то шаттл предполагался частью STS где был ядерный буксир на NERVA, и он то ужо потом мог в марс.
Аноним 01/06/19 Суб 14:34:01 486975207
>>486973
> А тебе обязательно без внешнего бака?
Да, потому что топливо из внешнего бака полность расходуется для выхода на НОО.
Аноним 01/06/19 Суб 15:12:04 486979208
>>486975
>Да, потому что топливо из внешнего бака полность расходуется для выхода на НОО.
Тебе что ли в однопуск обязательно? Тогда да, хрен тебе ни луны ни марса. Даже ядерный буксир (если буксировать сам шаттл) не поможет.
Аноним 01/06/19 Суб 15:13:41 486980209
>>486979
> Тебе что ли в однопуск обязательно?
Да, моя фантазия основана на этом.
> Тогда да, хрен тебе ни луны ни марса.
Разве 20 тонн топлива не хватит слетать до Луны?
Аноним 01/06/19 Суб 15:45:00 486987210
>>486980
>Разве 20 тонн топлива не хватит слетать до Луны?
Неправильная постановка вопроса, надо про дельту интересоваться, т.к. "количество топлива" бессмысленно без удельного импульса и сухой массы. А вообще до низкой лунной орбиты примерно столько же нужно, сколько и до марсианской, чуть поменьше.
Аноним 01/06/19 Суб 17:35:56 487022211
>>486980
>> Тогда да, хрен тебе ни луны ни марса.
>Разве 20 тонн топлива не хватит слетать до Луны?
Не хватит.

Для того что бы на 20 тоннах топлива на шаттле слетать до луны нужен двигатель более эффективный чем нерва2.0 (тимберлейк) который рисовали в маня 80-х, а тот был тот еще маня-огого.

Но в два пуска (40тонн "топлива" условно) на луну норм. В три пуска на марс норм. Без возврата. Ну и водород все равно столько не хранится даже сейчас.
Аноним 01/06/19 Суб 17:37:30 487023212
>>487022
Да, это все естественно для NTR (ядерной тепловой ракеты), проект нерва-тимберлейк, вся фигня
Аноним 01/06/19 Суб 17:39:22 487024213
>>486987
>т.к. "количество топлива" бессмысленно без удельного импульса и сухой массы.
Он дал начальные условия норм вообщето - летим на спейсшаттл орбитере, используя грузовой отсек для доп топлива и оборудования.
Аноним 01/06/19 Суб 17:45:28 487026214
>>487022
Собственно продолжая бред - сначала один пуск ДелтаХеви, водород храним в доработаном центавре(верхняя ступень). хз могут понадобится доп резервуары для водорода Потом запускаем шаттл с ядреным движком. Шаттл стыкуется с центавром, ядерный движок поднимается из грузового отсека и жжет тягой. Центавр с доп баками после израсходования водорода отцепляем.
Аноним 01/06/19 Суб 17:51:36 487029215
Раз до Луны не долететь то куда максимально можно, может то ГСО?
Аноним 01/06/19 Суб 17:53:22 487030216
>>487029
Спечётся он на ГСО от солнышка. Да и хули там делать? Радиационный курорт для туристов устраивать? Это ж прямо посередине внешнего радиационного пояса.
Аноним 01/06/19 Суб 17:56:18 487031217
>>487030
Печальную реальность оставим за скобками, меня интересует теоретическая возможность.
Аноним 01/06/19 Суб 17:58:54 487033218
>>487031
Ну теоретически-то с баками в трюме у него хватит дельты до ГСО, да.
Аноним 02/06/19 Вск 11:33:28 487145219
iTdoM3Fe0nc.jpg (122Кб, 800x800)
800x800
Возможно ли объяснить у некоторой выдуманной планеты наличие небольших областей с ускорением свободного падения в несколько раз большим, чем на остальной поверхности планеты? Тип если представить, то допустим на планетке будет 1,5 м/с^2, а на трёх с половиной пятачках (по площади, относительно ко всей площади поверхности планеты) будет 10 м/с^2. Я вот думал, мол, если у планетки будет ядро вытянутое, как пельмень, а ещё если прямо аккуратно над складочками этого пельменя будет тонкая кора, то может быть и можно будет получить такие перепады. Есть ли ещё какие-нибудь варианты? Заранее спасибо, кст, чё там с вазимиром бля?
Аноним 02/06/19 Вск 11:40:31 487146220
Аноним 02/06/19 Вск 11:55:12 487150221
>>487145
>Возможно ли объяснить у некоторой выдуманной планеты наличие небольших областей с ускорением свободного падения в несколько раз большим, чем на остальной поверхности планеты?
Что такое небольших. Если у тебя планета крутится быстро, то на экваторе будет нормас, а на полюсах будет плющить. Есть классическая НФ про живущих в таких условиях.

Схема с пельменем работает только для очень маленьких тел, вроде изначального пельменя Пана. У них и ядра-то не может быть в классическом смысле. На таких телах и гравитация будет мизерной, пукнешь и улетишь нахуй. Есть ещё масконы в коре, но они во-первых не дадут настолько сильных вариаций, а во-вторых выравняются при сколько-нибудь сильной гравитации тела.

Так что нет, чем больше тело, тем больше его форма определена самогравитацией. Всё что имеет хотя бы 1g плюс-минус сапог, будет очень равномерным внутри. И чем больше самогравитация, тем более равномерным - нейтронные звезды например почти идеально шарообразные из-за этого. Единственный способ получить разную гравитацию на поверхности - раскрутить шарик. (но и у этого есть предел, твою планету распидорасит задолго до достижения невесомости на экваторе, ибо ты по сути контришь раскруткой самогравитацию, которая удерживает его от распидораса, начиная с некоторого значения)
Аноним 02/06/19 Вск 11:59:45 487151222
>>487145
>кст, чё там с вазимиром бля?
Нихуя, заглохло все
Аноним 03/06/19 Пнд 09:44:11 487325223
b9ee36165411a12[...].jpg (91Кб, 400x400)
400x400
>>487150
НФ:
Хол Клемент - Экспедиция «Тяготение»
Вполне годный роман, очень легко читается, рекомендую
Аноним 03/06/19 Пнд 11:02:47 487329224
B2lforf2bi4.jpg (118Кб, 600x563)
600x563
А что, разве у SW1149 сферическое зеркало? Вроде в гугле пишут что параболическое. Где правда?
Аноним 03/06/19 Пнд 11:44:22 487333225
Я думаю многие задумывались о том как может выглядить разумная форма жизни. Если так подумать, то получается, что они скорее всего тоже будут гуманоиды? Так же с двумя руками и ногами.
Я недавно навернул "Прибытие" и задался вопросом, а как инопланетяни построили корабль? У них там и стекло от защиты, и корабль весь такой ровный, а ведь пальцев и подходящих конечностей у них ведь нет. То есть понятно, что инопланетян хотят показать как можно более не похожих на нас, но что если это не верно с биологической точки зрения?
Разве эволюция может наделить разумом существо не могущее построить оружие, и вообще работать с мелкими и крупными деталями, а тем более прогрессировать до космических полётов?
Может всё таки как это не казалось банально, но инопланетяне (именно разумные и способные в технологический прогресс) будут всё таки гуманоиды и близко похожие на нас, как в фильмах Прометей и Аватар, а не кальмары или пауки бороздящие космос, что думаете?
Аноним 03/06/19 Пнд 11:45:39 487334226
sketch-15595510[...].png (157Кб, 720x1280)
720x1280
>>487333
Я кстати только что задал этот вопрос в /b/, в треде про космос, намекая, что именно приматы в первую очередь всё таки идеальны для эволюции (как мы собственно видим). Но со мной начали спорить и говорить, что это не имеет значения, тупо рандом и что даже муравьи и слоны разумные.

Вы же тут люди посерьёзнее, поясните если я не прав. Может тут у вас и эволюционные биологи даже есть
Аноним 03/06/19 Пнд 12:26:37 487339227
>>487333
>>487334
В сайфаче эта тема многократно обоссывалась ефремофагами. Основной посыл - да, для технологического и цивилизационного развития у инопланетян должны быть сложные рабочие конечности, поэтому то или иное конвергентное сходство с земными приматами неизбежно.
Аноним 03/06/19 Пнд 13:05:54 487342228
Есть ли предел у расширения вселенной и каким он будет?
Я так понимаю расширение происходит благодаря растягиванию пространства/материи (я хз точно), но ведь это не может вечно быть. Может ли так быть, что она начнёт истончаться и рваться в итоге?
Может хуйню говорю, короче: как произойдёт смерть вселенной, теоретически?
Аноним 03/06/19 Пнд 14:06:39 487347229
image.png (46Кб, 200x200)
200x200
>>487342
>но ведь это не может вечно быть.
может
>Может хуйню говорю, короче: как произойдёт смерть вселенной, теоретически?
все планеты и звезды остынут. протоны развалятся и материя будет буквально рассыпаться. останутся черные дыры и фотоны которые летят в космосе бесконечно. потом черные дыры испарятся, фотоны потеряют энергию. не считая рандомных флуктуаций во вселенной не будет ни молекулы.

Абсолютно пустое пространство которое бесконечно расширяется. Понятие "время" потеряет смысл - им нечего будет мерять поскольку ничего не происходит и не меняется. И так будет бесконечно долго - никогда ничего больше не произойдет в абсолютно темной абсолютно пустой вселенной.
Аноним 03/06/19 Пнд 14:17:41 487349230
Аноним 03/06/19 Пнд 15:56:13 487355231
>>487333
Тащемта только в человеке есть божья искра.
Аноним 03/06/19 Пнд 16:00:40 487356232
>>487347
Очень огорчает такая перспектива, хотя она и далёкая.
Аноним 03/06/19 Пнд 17:32:26 487360233
Аноним 03/06/19 Пнд 17:33:56 487361234
Аноним 03/06/19 Пнд 23:08:59 487403235
>>487347
>протоны развалятся
Це ж только гипотеза. Никто не наблюдал распад протона.
Аноним 04/06/19 Втр 09:36:30 487449236
>>487403
Распад ксенона тоже никто не ожидал

>Ученые зафиксировали одно из самых редких физических явлений — особый тип радиоактивного распада в ксеноне-124. Период полураспада ксенона, то есть время, за которое распадается половина вещества, достигает 1,8 умноженное на 10 в 22-й степени лет, то есть в триллион раз превышает возраст Вселенной.
Аноним 04/06/19 Втр 09:39:37 487451237
15158439647690.png (112Кб, 500x500)
500x500
>>487403
>Никто не наблюдал распад протона
Ну это с какой стороны посмотреть.
Аноним 04/06/19 Втр 10:45:12 487454238
>>487449
Нихуясе. Это значит, что если мы будем просто хранить обычный ксенон, то у нас будут постоянно небольшие потери?
Аноним 04/06/19 Втр 10:49:21 487455239
>>484908 (OP)
>Что будет с человеком если он попробует ступить на поверхность газового гиганта?
У ГГ есть поверхность на которую можно ступить?
Аноним 04/06/19 Втр 10:58:46 487456240
Аноним 04/06/19 Втр 12:40:45 487484241
>>487449
Но протон-то пока стабилен. Не засекли распад еще. Если он стабилен, то что тогда в итоге?
Аноним 04/06/19 Втр 12:56:17 487489242
>>487484
>Не засекли распад еще.
про ксенон до 2019 так же говорили и где он теперь?
Аноним 04/06/19 Втр 14:00:54 487492243
>>487489
Но про протон-то не говорят
Аноним 04/06/19 Втр 14:08:24 487493244
>>487492
двумя постами выше говорят
Аноним 04/06/19 Втр 14:22:23 487494245
>>487493
Причем тут твоя ракета?
Аноним 04/06/19 Втр 14:25:43 487495246
сложно
Аноним 04/06/19 Втр 15:02:48 487496247
>>487494
При том что я тебе ее за щеку сунул
Аноним 04/06/19 Втр 23:30:55 487542248
>>487454
Да, через септиллион лет у тебя останется половина!
Аноним 04/06/19 Втр 23:36:07 487543249
>>487542
Ну хоть что-то. На старость хватит.
Аноним 05/06/19 Срд 08:53:46 487564250
Аноним 05/06/19 Срд 11:36:09 487591251
Аноним 05/06/19 Срд 12:07:20 487599252
>>487591
Что там, в крации?
05/06/19 Срд 12:10:20 487600253
>>487599
Очередной высер на ютюбчике.
Аноним 05/06/19 Срд 12:12:26 487602254
>>487599
Рассказывает почему именно у нас, как у вида, появился разум. У Дробышевского подача материала очень хорошая, легко слушается, ты начни и не заметишь как уже всю лекцию прослушал. Мы его компанией человек в 4-5 обычно смотрим, хорошо заходит.
Аноним 05/06/19 Срд 14:39:46 487622255
>>487564
Если какая-то твердая поверхность и есть, то только у каменного ядра в тысячах километров под видимой «поверхностью». Но инфа не 100%, внутрь газовых гигантов никто не заглядывал.

Во внешних слоях ничего твердого нет, там в основном только водород, плавно переходящий из газа в жидкий металлический.
Аноним 05/06/19 Срд 14:45:20 487623256
>>487622
Там вроде сверхкритика в металлический водород переходит.
Аноним 05/06/19 Срд 20:33:36 487669257
Аноним 05/06/19 Срд 20:39:30 487670258
rd-180flowdiagr[...].jpg (61Кб, 1123x702)
1123x702
>>484908 (OP)
Почему Охлаждение Кс ЖРД никогда не делают жидким кислородом, даже если это единственный криогенный компонент?
Аноним 05/06/19 Срд 21:03:29 487675259
>>487670
Потому что окислитель
Аноним 05/06/19 Срд 22:16:39 487685260
>>487622
Чому у юпитера столько разных цветов? Это всякие аммиаки, водяной пар, метан и тд?
Аноним 05/06/19 Срд 22:18:32 487686261
Алсо можно ли скажем зависнуть в атмосфере Юпитера на аэростате на такой высоте, где давление равно земному и при этом выйти наружу без вреда для здоровья? В подогреваемом герметичном костюме с подачей кислорода естественно.
Аноним 05/06/19 Срд 22:20:50 487687262
>>487686
> и при этом выйти наружу без вреда для здоровья? В подогреваемом герметичном костюме с подачей кислорода естественно.
Радиация быстро сделает в этом скафандре котлету по-киевски.
Аноним 05/06/19 Срд 22:22:47 487688263
>>487687
У юпитера мощная магнитосфера вроде как.
Аноним 05/06/19 Срд 22:50:20 487691264
>>487687
В радиационных поясах да. Но не в атмосфере, под ними.

>>487686
Нельзя, атмосфера водородная. Чем бы ты не надул шар - будешь тонуть, пока тебя не расплющит давлением. Любой летательный аппарат на Юпитере будет тяжелее воздуха. Летают там лишь аэрозоли и прочие дисперсные системы.

Кроме того, в атмосфере газовых гигантов дуют ебанутые ветра, на Нептуне вообще сверхзвуковые, на Юпитере попроще, но и их более чем хватит чтобы распидорасить твоего незадачливого испытателя в бесконечном шторме, вместе с шаром. Плюс на Юпе 2.5g - не просто жмёт сильно и тонешь быстрее, но и все аэродинамические процессы происходят быстрее.
Аноним 05/06/19 Срд 22:51:51 487693265
>>487455
Нет.

>>487622
>Если какая-то твердая поверхность и есть, то только у каменного ядра в тысячах километров под видимой «поверхностью».
Задолго до неё металлический водород, и он не жидкий, а твердый. Так что переход в любом случае плавный.
Аноним 05/06/19 Срд 22:52:30 487694266
>>487691
Можно вакуумом надуть сделать сверхпрочный дирижабль, который будет держать форму, даже если внутри давление меньше, чем снаружи.
Аноним 05/06/19 Срд 22:54:16 487696267
Аноним 05/06/19 Срд 22:55:01 487697268
Аноним 05/06/19 Срд 22:55:26 487698269
>>487691
Ок, сделать вертолет для юпитера? А насчет ветра - так лететь просто вместе с ветром. Там же обширные зоны перемещающегося газа.
Аноним 05/06/19 Срд 22:56:22 487699270
>>487694
Теоретически можно, практически нельзя.
Аноним 05/06/19 Срд 22:56:23 487700271
Аноним 05/06/19 Срд 22:57:56 487701272
>>487698
Аппарат тяжелее воздуха - всегда пожалуйста. Но ему тоже пизда.
>А насчет ветра - так лететь просто вместе с ветром. Там же обширные зоны перемещающегося газа.
Даже эти обширные зоны в штиле никогда не бывают, дикая турбулентность там абсолютно везде.
Аноним 05/06/19 Срд 22:59:19 487702273
>>487699
Ну с современными технологиями наверняка. Но ведь по сути своей подводная лодка - это как раз такой дирижабль, только водоплавающий. Он, конечно, тяжелый что пиздец, но хуй знает, на что способны йоба-графены и прочая еще непридуманная муть.
Аноним 05/06/19 Срд 23:00:24 487704274
>>487670
Помимо уже упомянутой коррозии, у кислорода еще и так себе с теплоемкостью, у керосина и метана она более чем вдвое выше, у водорода — в 10+ раз.
Аноним 05/06/19 Срд 23:03:03 487705275
>>487691
Еще помимо дирижабля есть вариант с подогретым водородом. Собственно, многие воздушные шары так и летают.
Аноним 05/06/19 Срд 23:08:38 487706276
>>487693
>Задолго до неё металлический водород, и он не жидкий, а твердый
Именно что жидкий. Для твердого там горячо слишком.

https://science.nasa.gov/science-news/science-at-nasa/2011/09aug_juno3
>Jupiter's strangest feature, however, may be a 25,000 mile deep soup of exotic fluid sloshing around its interior. It's called liquid metallic hydrogen.
Аноним 05/06/19 Срд 23:12:15 487708277
1520258711054.jpg (34Кб, 496x600)
496x600
>>487706
>Click on the image to launch a ScienceCast video about the mysteries of Jupiter's interior.
>ссылка ведет на jpg скрин видеоплеера с кнопкой Play
Ну еб их мать
Аноним 05/06/19 Срд 23:15:27 487709278
>>487706
Это на поверхности. Ниже следует фаза сверхтекучего твердого тела.
Аноним 05/06/19 Срд 23:17:23 487710279
>>487709
>>487706
>>487693
>>487622
там и не жидкость, и не твердое тело, а несколько экзотических состояний, не похожих ни на то, ни на другое при нормальных условиях
Аноним 05/06/19 Срд 23:28:41 487713280
Кот.jpg (11Кб, 196x194)
196x194
>>487709
Чиво? Сверхтекучесть бывает только тогда, когда энергия частиц падает настолько, что квантовые эффекты начинают играть заметную роль, т.е. около абсолютного нуля, а внутри Юпитера много тысяч кельвинов. Ты явно перепутал со сверхкритической жидкостью (в которой от твердого тела нет ничего).

>>487710
Скорее там что-то между жидкостью и газом, упорядоченности твердого тела там явно нет.
Аноним 05/06/19 Срд 23:45:28 487714281
>>487713
>Сверхтекучесть бывает только тогда, когда энергия частиц падает настолько, что квантовые эффекты начинают играть заметную роль, т.е. около абсолютного нуля
Или при гигантском давлении. https://arxiv.org/abs/1702.00211
>упорядоченности твердого тела там явно нет
Сверхтекучее твердое тело (supersolid) это и есть упорядоченная решетка, проявляющая свойства сверхтекучести при этом. Вот такая экзотика, хотя казалось бы одно несовместимо с другим. При таких давлениях обычных соединений не бывает. спроси Оганова, он знает.
Аноним 05/06/19 Срд 23:49:40 487715282
>>487713
>Ты явно перепутал со сверхкритической жидкостью (в которой от твердого тела нет ничего).
В ней и от жидкости не так много как кажется. Но суперфлюиды сильно выше по фазовой шкале, чем металлический водород.
Аноним 06/06/19 Чтв 01:04:38 487717283
У меня есть вопрос по астрологии.

НЕТ, УБЕРИТЕ РУКИ, Я ПО АДРЕСУ ЗАШЕЛ.

Так вот, способствовала ли астрология когда-нибудь развитию астрономии, например, открытием новых звезд или законов, или всегда сидела на шее и была абсолютно полностью бесполезным куском говна?
Аноним 06/06/19 Чтв 01:06:27 487718284
>>487691
Ебать, Юпитер страшный.
Аноним 06/06/19 Чтв 02:46:03 487723285
>>487717
>сидела на шее и была абсолютно полностью бесполезным куском говна
Думаю, что это. Зачем астрологам новые небесные тела? Им от таких открытий только проблемы одни--внезапно нужно придумывать как "учесть" "влияние" каких-то новых фигнюшек.
Аноним 06/06/19 Чтв 03:25:20 487730286
>>487717
Опосредованно, позволяла легко выбивать деньги на изучение движения небесных тел и объяснять быдлу зачем тебе все эти дорогие железки.
Аноним 06/06/19 Чтв 03:35:31 487731287
2.png (8Кб, 467x236)
467x236
Если в центре звезд невесомость, то за счет чего проиходит синтез в ядре?
Аноним 06/06/19 Чтв 04:10:53 487733288
>>487731
За счет давления, создаваемого вышележащими слоями. То, что вещество в самом-самом центре нихуя не весит и никакого вклада в обжатие ядра не вносит, никак не отменяет того, что все снаружи от него весит дохуя.
Аноним 06/06/19 Чтв 11:44:55 487780289
Если за пределами вселенной нет времени и пространства, то получается, что человечество так и исчезнет ничего не узнав? То есть мы обречены ничего не познать?
Если пределы вселенной физически никогда покинуть не сможет, то как мы узнаем, есть ли например мультивселенная? Возможно всё, что нас окружает это лишь малая часть чего-то большего.

Что вообще наука говорит о том, что за пределами вселенной и можно ли нам посмотреть когда нибудь что там?
Аноним 06/06/19 Чтв 12:16:13 487788290
>>487691
>сверхзвуковые
>ветра
Што
Аноним 06/06/19 Чтв 12:18:02 487790291
>>487780
>То есть мы обречены ничего не познать?
Ща ебанут техническую сингулярность и все узнаем.
Аноним 06/06/19 Чтв 12:21:56 487791292
>>487780
Ничего не говорит наука.
Никаких гипотез об этом строить нельзя, т.к. пока нет идей как что-то такое можно проверить.

Наука - не религия где ответы на все вопросы (и обычно в виде бога дыр), в науке можно сказать "я не знаю".

И ты можешь сейчас представить себе, что пока нет никаких оснований считать, что есть что-то за пределами вселенной, вся вселенная - это все что есть, больше ничего нет.
Я знаю, про пятно, но оно до сих пор ничего не доказало.
Аноним 06/06/19 Чтв 12:22:18 487792293
>>487788
Ветер, скорость которого превышает скорость звука. Проблемс?
Аноним 06/06/19 Чтв 13:43:44 487806294
>>487791
Гипотезы можно строить какие угодно. Другое дело, что пока не появится способа их проверки, ничего определенного об этом сказать нельзя. Тут ситуация хуже, чем с внутренностью черной дыры - если особо любопытный может упасть в дыру и проверить гипотезы о ней самостоятельно, то в отношении вселенной неизвестно даже способа ее покинуть.
Аноним 06/06/19 Чтв 15:18:17 487833295
>>487792
Интересна физика процесса.
У нас тут на землящке посоны всякие хитрозаебущные форсунки и эжекторы да сопла изобретают, чтоб получить сз газовый поток, а там ветер дует прост потому что иди нахуй вот почему?
Аноним 06/06/19 Чтв 15:27:51 487834296
>>487704
Это с учётом скрытой теплоты испарения?
Аноним 06/06/19 Чтв 15:42:16 487835297
>>487788
Наиболее динамично развивающиеся ветры в Солнечной системе дуют на Нептуне — их скорость порядка 2400 км/ч
Аноним 06/06/19 Чтв 20:23:37 487876298
>>487834
Нет, это просто теплоемкость. Кипения в охлаждающей рубашке вообще стараются не допускать, так как если образуется пар, то теплопередача через него сразу резко падает, и стенка может перегреться и прогореть.
Аноним 07/06/19 Птн 00:30:41 487908299
>>487833
Да просто большие БОЛЬШИЕ градиенты температур и давлений, вот и всё. Это же планета-гигант.
Аноним 07/06/19 Птн 09:27:35 487930300
>>487691
>Любой летательный аппарат на Юпитере будет тяжелее воздуха
пездеж. шар наполненный вакуумом
Аноним 07/06/19 Птн 09:29:33 487931301
>>487930
> шар наполненный вакуумом
Жидким надеюсь?
Аноним 07/06/19 Птн 09:38:26 487933302
image.png (12Кб, 898x45)
898x45
>>487806
>Гипотезы можно строить какие угодно. Другое дело, что пока не появится способа их проверки, ничего определенного об этом сказать нельзя.
Нет, если твоя задумка непроверяема - это не научная гипотеза.
Аноним 07/06/19 Птн 09:38:59 487934303
>>487835
>км/ч
У нас спейсач или где?
Используй СИ.
Аноним 07/06/19 Птн 11:39:59 487950304
>>487933
Маня, способ проверки теориии - сюрприз! сюрприз! - диктуется самой теорией, и пока ты ее не построишь, ничего о ее проверяемости/непроверяемости ты сказать не сможешь.

А вообще, есть эмпирическое правило - если кто-то в дискуссии на бордах первым вспоминает про критерий Жоппера, то он с вероятностью, близкой к 100%, есть чухан, говноед, стукач и провокатор.
Аноним 07/06/19 Птн 11:41:30 487951305
Стикер (0Кб, 512x512)
512x512
>>487950
>если кто-то в дискуссии на бордах первым вспоминает про критерий Жоппера, то он с вероятностью, близкой к 100%, есть чухан, говноед, стукач и провокатор.
Заметьте, я про него ничего не говорил, я лишь формальное определение гипотезы дал, а вот ты его первый упомянул.

Аноним 07/06/19 Птн 11:42:23 487952306
>>487950
И да, какие нахуй теории, если речь шла о гипотезах? Ты даже эти термины путаешь?
Неудивительно, что ты сам под свои определения и попадаешь.
Аноним 07/06/19 Птн 12:48:36 487960307
>>487951
>>487952
>я про него ничего не говорил ррряяяяяя!!!1
Оно у тебя на приклейтеде, чухоня. И не бугурти ты так, аж два поста высрал ))))
>какие нахуй теории, если речь шла о гипотезах
Я говорю в терминах теоретической физики (где любой пук+вскукарек считается теорией), а не в терминах википедальной школоты.
Аноним 07/06/19 Птн 12:51:13 487961308
image.png (55Кб, 225x225)
225x225
>>487960
Всё с тобой понятно. Можешь продолжать, это забавно.
Аноним 07/06/19 Птн 13:16:28 487963309
1559362867195.jpg (342Кб, 1510x1148)
1510x1148
>M87zoom.webm
Как так зумируют вообще?
А можно ли сделать такой же, но чтобы смотреть микробов на жопе человека на другом континенте?
Аноним 07/06/19 Птн 13:16:58 487964310
Аноним 07/06/19 Птн 13:21:20 487965311
>>487963
Там не зум с одной камеры, а разные камеры, разные зумы, спектры.
Аноним 07/06/19 Птн 14:16:13 487975312
>>487950
>чухан, говноед, стукач
А вот и биомусор из /b/ подтянулся.
Аноним 07/06/19 Птн 14:38:19 487978313
badumtss.png (413Кб, 783x776)
783x776
>>487961
>>487975
Подрывом википедальных школотунов удовлетворен:3
Аноним 07/06/19 Птн 14:41:11 487979314
двачую
Аноним 07/06/19 Птн 18:44:44 488023315
Только сегодня узнал реальную причину невесомости на МКС. Тогда сразу тупой вопрос: насколько невесомость дальше от земли будет отличаться от невесомости на станции? Скажем, где-нибудь между Марсом и Землей. Без всякого ускорения. Просто прилетел туда и вышел из корабля в открытое странствие.

Если точно такие же свойства передвижения будут, то почему?

Аноним 07/06/19 Птн 19:07:21 488026316
>>488023
потому что там будет орбита вокруг солнца
Аноним 07/06/19 Птн 19:11:09 488027317
>>488026
Параметры гравитации, свободного падения и прочего не отличаются нигде в космосе?
Аноним 07/06/19 Птн 19:17:23 488029318
Аноним 07/06/19 Птн 19:18:17 488030319
>>488029
Кстати, почему я один только вставляю ссылки на русском языке без %АУЕ%ХУЙ%228 и т.п.?
Знаю, что нерилейтед, но почему до сих пор ссылки корежатся у людей?
Аноним 07/06/19 Птн 19:20:40 488031320
Я не понел, почему если запустить с орбиты земли аппарат на очень вытянутую орбиту (скажем чтобы она пересекала орбиту юпитера), то аппарат все равно вернется к земле если не окажется близко к юпитеру или типа того? Ведь гравитационное поле земли на таком расстоянии вообще неочем и почти не влияет на космический аппарат, даже солнце влияет больше. Почему он не полетит просто дальше по параболической траектории и не станет спутником солнца, а непременно вернется в периапсис земли?
Аноним 07/06/19 Птн 19:22:25 488033321
Аноним 07/06/19 Птн 19:22:42 488034322
>>488030
Не ты один. Вообще эти проценты это стандарт https://en.wikipedia.org/wiki/Percent-encoding и 99% браузеров кодируют урлы в такую еботу по хорошей причине. Просто не все знают что нужно поставить пробел в начале, чтобы отказаться от кодировки.
Аноним 07/06/19 Птн 19:25:33 488037323
>>488031
Если будет пересекать орбиту Юпитера - не вернется. Орбита должна быть внутри зоны гравитационного влияния (т.е. влияние тела должно пересиливать иные возмущения), иначе не вернешься.
Аноним 07/06/19 Птн 19:27:26 488038324
>>487963
Можно, снимаешь последовательно кучу картинок всё ближе к жопе, а финальные кадры вообще микроскопом. Склеиваешь это все в один видеоряд и получаешь микробов с другого континента. https://www.youtube.com/watch?v=INqKqk-ifpc
Аноним 07/06/19 Птн 19:30:01 488039325
>>488037
Что значит должно пересиливать? А если влияние других тел такое же незначительное, как влияние земли? Вообще странно, симуляторы всегда показывают, что как бы вытянута орбита ни была в пределах возможностей симулятора, периапсис всегда находится в точке старта. Видимо не учитывается влияние других тел
Аноним 07/06/19 Птн 19:56:43 488042326
>>488039
значит что тело начнет притягивать аппарат настолько сильно, что он изменит орбиту или вообще останется на орбите нового тела
>симуляторы всегда показывают, что как бы вытянута орбита ни была в пределах возможностей симулятора, периапсис всегда находится в точке старта
потому что надо именно неподалеку пролететь, чтобы было влияние. Например даже в КСП орбита меняется после близкого пролета иного тела
Аноним 07/06/19 Птн 20:06:27 488043327
5-Figure4-1.png (44Кб, 928x464)
928x464
>>488039
То и значит. На определенном расстоянии от тела влияние какого-нибудь другого тела уже будет сильней. Причем это расстояние не всегда одинаково, вот пролетит планета поближе и захватит спутник на высокоэллиптической орбите другой планеты, который как раз к ней поближе подошел. На пертурбациях низкоскоростных участков орбиты основана куча хитровыебанных низкоэнергетических траекторий, например баллистический захват, позволяющий магически превратить высокоэллиптическую орбиту вокруг одного тела в круговую вокруг другого.

Вокруг любого тела есть сфера Хилла, она же сфера гравитационного влияния, которая на самом деле не сфера, а полость немного неправильной формы с постоянно изменяющимися границами, т.к. соседние тела "теснят" её и движутся относительно неё. Внутри сферы Хилла доминирующей силой является гравитация тела, и спутник всегда движется по замкнутой орбите, если не превышает вторую космическую скорость. (в свою очередь падающую с дистанцией до тела). На границе сферы Хилла вторая космическая скорость обращается в 0 (как в точке Лагранжа), пукнешь и улетишь.

>Вообще странно, симуляторы всегда показывают, что как бы вытянута орбита ни была в пределах возможностей симулятора, периапсис всегда находится в точке старта
Это какие? То что ты описываешь, выглядит будто симулятор тебе рисует оскулирующую (кеплерову) орбиту, или вообще пиздит. В орбитере, например, он во многих случаях не учитывает вторую космическую. В любом случае неважно, это все предсказание и способ рисования в конкретной программе.

Рекомендую поломать себе мозг об GMAT https://sourceforge.net/projects/gmat/ , выполни тамошние туториалы хотя бы, и очень многие вопросы по орбитальной механике отпадут. По эллипсам аппараты летают только в учебниках.
Аноним 07/06/19 Птн 20:16:02 488046328
>>488031
>Я не понел, почему если запустить с орбиты земли аппарат на очень вытянутую орбиту (скажем чтобы она пересекала орбиту юпитера), то аппарат все равно вернется к земле если не окажется близко к юпитеру или типа того? Ведь гравитационное поле земли на таком расстоянии вообще неочем и почти не влияет на космический аппарат, даже солнце влияет больше. Почему он не полетит просто дальше по параболической траектории и не станет спутником солнца, а непременно вернется в периапсис земли?
Так ведь не вернется. Тебе надо очень тонко расчитать чтобы возвратная солнечная орбита пересекала земную притом в тот момент когда там находится земля.
В космосе тебе надо думать в четырех координатах, еще и время учитывать. Кербач в этом плане отлично помогает, как и ранние видео Скотта "Наше Всё" Мэнли.

>>488034
Я без пробела. В стандартном хромоге надо пробел, а в лучшей опере (которая Вивальди) - такого нет, сразу искаропки делает.
Аноним 07/06/19 Птн 20:18:46 488047329
>>488046
>а в лучшей опере (которая Вивальди) - такого нет, сразу искаропки делает
Она искаропки нарушает стандарты, они не зря придуманы. Но хуй с тобой, здесь не /s/ все же.
Аноним 07/06/19 Птн 20:26:35 488050330
>>488043
>баллистический захват, позволяющий магически превратить высокоэллиптическую орбиту вокруг одного тела в круговую вокруг другого.
А насколько долго такой захват может продолжаться? Есть же какой-то механизм который и позволил захватить спутники в круговые орбиты, а не вышвыривать их на пятом проходе.
Дай угадаю: время? Захвачены были миллионы, уселись единицы, удачно попав в синхрон, это так?

>>488047
У меня не один бровзер, я выбрал для серфа тот, который как раз очень удобно делает все вообще.
Не та самая Опера, что раньше, но ближе всего к ней.
Сейчас гугель намудрыжит с движком и окончательно к лисам перееду, че тут думать. Это не религия для меня, как для некоторых, есть что лучше - буду юзать.
Аноним 07/06/19 Птн 20:45:32 488058331
Dawn-navigation[...].png (145Кб, 870x640)
870x640
20160318ssc2009[...].jpg (154Кб, 1500x1200)
1500x1200
>>488050
>А насколько долго такой захват может продолжаться?
Ну мы же об искусственных спутниках-то говорим вроде, поэтому быстро, если правильно рассчитать. Dawn при подходе к Церере если не ошибаюсь использовала баллистический захват, управилась быстро. Да и многие другие спутники так летали. Но да, конечно, любая низкоэнергетическая траектория это всегда размен дельты на время. Все интересные процессы в таких траекториях происходят на очень медленных участках траектории, например в случае баллистического захвата нужно чтобы и без того малая пертурбация оказалась больше чем текущая орбитальная скорость.

Вообще, у людей незнакомых с орбитальной механикой низкоэнергетические траектории ломают шаблон обычно потому, что они плохо представляют как спутник движется в разных точках отсчета. Это реально не интуитивно и способно по первости взорвать мозг, потому я и посоветовал GMAT, это первое чему там учат в туториале, очень наглядно.

>Захвачены были миллионы, уселись единицы, удачно попав в синхрон, это так?
Ну в случае естественных спутников так и есть, например Феба скорее всего так и была захвачена Сатурном (а затем частично эродировала в кольцо), уж очень необычная траектория, как раз во внешней части SOI Сатурна.
Аноним 07/06/19 Птн 21:08:21 488066332
Без названия.jpg (302Кб, 2743x2400)
2743x2400
Почему просто нельзя тупо поставить линзу(или несколько) из полимеров на L1 пару квадратных км3 и ещё приемники-ретрансляторы на геостационарке, что бы пучек бил в огромную. черную железную бочку с солью и водяным контуром 24/7

Можно ведь запросто 50-100 тераватт поднять

щит 1000км2 на Л1 может убрать 1% излучения(тыщи террават в сутки)

этож блядь гениально

Возможно полотна из полимера 100 на 100 метров хватит на 10 Россий

а че будет с атмосферой если такой лазир её будет прорезать круглый год?
Аноним 07/06/19 Птн 21:10:27 488067333
Аноним 07/06/19 Птн 21:11:52 488068334
Аноним 07/06/19 Птн 21:15:18 488069335
>>488068
А прикинь, лазир такой пучеками света - БДЫЩ, БДЫЩ! этож блядь гениально
Аноним 07/06/19 Птн 21:19:00 488070336
14252463463.jpg (28Кб, 512x422)
512x422
>>488069
Но ведь л1 орбита стабильна(с двигателями) и гораздо ближе к солнцу, можно реально прислать на землю несколько сот тыщ магаватт по цене 2-3 атомных станций

Или атмосфера сломается?
Аноним 07/06/19 Птн 21:27:27 488071337
>>488050
>А насколько долго такой захват может продолжаться? Есть же какой-то механизм который и позволил захватить спутники в круговые орбиты, а не вышвыривать их на пятом проходе.
А, только сейчас понял суть вопроса. При идеальном совпадении сразу же и захватывается, без каких-либо дополнительных танцев. При неидеальном может и телепаться куда-нибудь. Но идеальное, как ни странно, более вероятно.
Аноним 07/06/19 Птн 21:28:29 488072338
>>488070

Прислать в квадрат 100 на 100 метров*
Аноним 07/06/19 Птн 21:30:38 488074339
>>488070
>гораздо ближе к солнцу
Ага, аж примерно на процент ближе Земли. Поэтому мощность излучения Солнца там будет в целых ~1,02 раза выше чем на Земле. Ах ну да, круглосуточно и без влияния атмосферы. Ну пусть со всеми поправками в 3 раза больше. Чтобы получить этот гигантский выигрыш, ты должен 1) доставить и развернуть эту некислую ебалу в L1; 2) ворочать ее так, чтобы фокусировать пучеки в нужную точку на вращающейся Земле; 3) постоянно корректировать орбиту движками, ведь L1 нестабильна. Что-то мне кажется, что оно того не стоит, проще засадить 3 кв. км. Земли солнечными батареями, чем ебаться с 1 кв. км паруса в 1,5 млн км. от Земли.
Аноним 07/06/19 Птн 21:45:49 488076340
>>488074

1000km2 тени на л1 блокирует 1% света на землю(тыщи террават в сутки(мы потребряем 70 в год)

Бить будем в отражатель на удобной геостациорарочке, а он уже будет работать по бочке, можно вообще импульсно, отражатели будут налетать на тераватный сгусток энергии раз в час нагревая ебический буфер размером со стадион до тыщ градусов

Это в сотни раз дешевле любой штуки на земле
Аноним 07/06/19 Птн 22:17:00 488077341
>>488076
>1000km2 тени на л1 блокирует 1% света
Далеко не 1%. Учитывая близость L1, можем пренебречь углом. Земля имеет радиус 6350 км, что дает площадь круга в 126 млн кв. км. Отсюда 1000 кв. км. это менее 0,001%.

Прикинь к тому же стоимость запуска этой хрени (100 кв. км будут весить ДОХУЯ, из чего бы ты их не сделал). Представь технические сложности создания линзы, ОЧЕНЬ точно держащей форму при таких размерах, ведь малейшее искажение - и свет пойдет не в приемник а сожжет Воронеж, или, в лучшем случае, улетит к Нептуну. Прикинь затраты топлива на корректировку орбиты этой дуры, ведь L1 нестабильна. Учти необходимость замены через полвека - ведь метеориты (при таких размерах столкновения неизбежны) и пыль будут дырявить, царапать и замутнять твою линзу.

>можно вообще импульсно
Ага, ну то есть ты еще вдовесок собрался ставить на свою йоба-линзу солнечные батареи, аккумулятор и излучатель.
Аноним 08/06/19 Суб 07:45:59 488109342
>>488074
Самая мякотка что это буквальный луч смерти в случае (не)сознательной потери фокусировки на приемнике.

Я за орбитальные солярки, тема неплохая, но у нас на планете вполне есть варианты качественно и не сря добывать энергию: ТОРИЙ (И уран).
Аноним 08/06/19 Суб 08:07:29 488111343
>>488074
>Ага, аж примерно на процент ближе Земли


Давайте тогда пояс из линз или зеркал максимально близко к солнцу на солнечную орбиту!

Луч будет импульсно бить по системе отражателей на орбите земли а они уже фокусировать на бочку с солью.
Аноним 08/06/19 Суб 08:55:03 488114344
Какова природа такой необычности у звезды Пшибыльского?
Аноним 08/06/19 Суб 12:38:08 488131345
ketmitthinking.png (2021Кб, 1860x2190)
1860x2190
Как работает радиация? Посмотрел сериал чернобыль и не понял.
Графит же вроде сам по себе не радиоактивный?
А что там радиоактивное было?
И вообще почему материя накапливает ридиацию, а не отталкивает, если радиация это просто лучи.
Я не понял.
Аноним 08/06/19 Суб 12:45:22 488132346
>>488066
>щит 1000км2
че блять? знаешь сколько будет это весить и сколько стоить вывод? к тому же эта хуйня за пару лет продырявится солнечным ветром и микрометеоритами.
Аноним 08/06/19 Суб 12:48:36 488133347
>>488131
Потому что изза ядерной реакции вылетают нейтроны. Они летят, врезаются в другие атомы (графита например) и прилипают к ядрам. От чего те становятся нестабильны и разваливаются постепенно, излучая нейтроны, гамма-излучение, бета-излучение и прочую парашу. Ты сам будешь излучать, если на тебя посветить нейтронами.
Аноним 08/06/19 Суб 12:49:55 488134348
>>488133
Нейтроны это же электричество, не?
Аноним 08/06/19 Суб 12:50:57 488135349
yobapalm.jpg (10Кб, 229x220)
229x220
>>488134
троллинг тупостью?
Аноним 08/06/19 Суб 12:52:05 488136350
>>488135
Ну я давно вроде читал что нейтроны бегают по проводам или что-то такое. Поясни пожалуйста.
Аноним 08/06/19 Суб 13:27:14 488140351
your brain.png (218Кб, 640x358)
640x358
Аноним 08/06/19 Суб 13:29:33 488141352
>>488140
Перепутал нейтроны с электронами, бывает.
А ведь материя состоит из атомов, В которых есть нейтроны и протоны - почему всё не радиоактивное?
Аноним 08/06/19 Суб 14:07:25 488146353
>>488131
Тебя конкретно поражающие факторы интересуют?

Бывает гамма-излучение, которым светятся радиоактивные материалы. Это тот же самый свет, только круче. Круче ультрафиолета, круче рентгена, круче неба и аллаха. Очень хорошо проникает во всё и ебёт живые клетки и ДНК. От неё только одна защита - толстый слой плотной хуйни, или отойти подальше от фонящего материала. Графит от реактора сам излучал гамму, ибо был облучен прямо в реакторе сильным потоком нейтронов. Это наведенная радиоактивность.

Бывают еще частички радиоактивного материала. Они проникают в виде пыли, накапливаются в грибах, растениях, животных, земле. Проглотишь таких дохуя, сожрешь гриб такой, и они будут облучать тебя изнутри. От пыли соответственно защита противопылевая (очки, респиратор и т.п.), ну и химическая иногда. В воздух была выброшена именно такая хуйня, и район накрыло именно оно, рыжий лес оттуда же стал рыжим.
Аноним 08/06/19 Суб 14:08:51 488147354
>>488146
А почему радиация так слабо повлияла на растения?
Аноним 08/06/19 Суб 14:16:23 488149355
>>488146
>рыжий лес оттуда же стал рыжим.
Чо это он рыжий?
Аноним 08/06/19 Суб 15:27:41 488162356
>>488149
Потому что стал рыжим. Читать не умеешь?
Аноним 08/06/19 Суб 15:29:29 488164357
>>488162
Охуенно объяснил, почему масло масляное? Потому что стало масляным.
Аноним 08/06/19 Суб 15:36:05 488165358
Дано: есть обитаемая планета с жидкой водой. Наибольшая часть ландшафта - пустыни с оазисами, оазисы небольшие, но многочисленные и относительно близко к друг другу расположенные. И это именно пустыни, с песчаными дюнами и всё такое. Так вот при каких условиях - размер планеты, её состав, тип родительской звезды, положение планеты вокруг этой звезды, тип орбиты, скорость вращения, угол наклона оси, наклон прецессии, наличие спутника и его тип - на этой планете ветрищ не будет и дюны не будут надувать ветров в оазисы? Я имею в виду не полное отсутствие ветра, а просто его слабость.
Аноним 08/06/19 Суб 15:42:28 488166359
>>488165
Ну, по идее, чем однороднее климат, тем слабее ветер. Отсюда я бы сказал, минимальный наклон оси, почти круговая орбита, равномерный рельеф без гор и океанов. Быстрое вращение, чтобы цикл день/ночь не давал больших перепадов температуры.
Аноним 08/06/19 Суб 15:44:46 488167360
>>488166
Еще, вероятно, плотная и толстая атмосфера поможет.
Аноним 08/06/19 Суб 16:12:30 488169361
>>488165
А ничего что дюны это следствие ветров?
Аноним 08/06/19 Суб 16:18:14 488171362
>>488169
Допустим, в ранние геологически годы было буйство ветров, но вот сейчас такого нет.
Аноним 08/06/19 Суб 16:20:12 488172363
>>488171
Так они рассосутся со временем, че ты несешь вообще
Сделай тогда оазисы на холмах только хуй знает как там вода будет тогда
Аноним 08/06/19 Суб 18:38:24 488183364
1523429334783.jpg (728Кб, 900x900)
900x900
А вот почему нельзя в невесомости долго находиться? Из-за вреда для костей только? Нельзя ли как-то что-то сделать с самими костями, создать таблетки, которые замедлять вымывание кальция из организма, вместо того чтобы создавать огромные структуры из вращающихся центрифуг? Если взвесить тяжесть изобретения лекарственного решения проблемы и тяжесть изобретения и конструкции центрифуги, то лекарства и таблетки гораздо легче создать, так ведь?
Аноним 08/06/19 Суб 18:40:54 488184365
>>488183
>гораздо легче создать
Это не программирование, где достаточно написать

void tabletka () { this->calcium_vymivanie_rate /= 10; }
Аноним 08/06/19 Суб 18:45:44 488185366
>>488184
Да это может быть не просто, но полезней и для многих других заболеваний костей, может иметь пользу на Земле даже. И всё равно это может оказаться проще, чем создавать огромные центрифуги для дальних полётов в космос. Знаю, что там еще мышечная деградация, но её вроде как упражнениями останавливают. Медицинское решение может быть также комплексным. Это могут быть таблетки таргетирующие печень, в дополнение к каким-нибудь экзотическим мерам вроде обогащения костей редкими изотопами кальция, которым сложнее принимать участие в реакциях организма.
Аноним 08/06/19 Суб 18:49:55 488186367
>>488185
>редкими изотопами кальция, которым сложнее принимать участие в реакциях организма
Изотопы достаточно тяжелых элементов практически не отличаются друг от друга по химическим свойствам. Даже у водорода (легче уже некуда) отличия между протием и дейтерием не такие уж большие.
Аноним 08/06/19 Суб 18:52:28 488189368
>>488186
ну, может быть. Я еще где-то читал, что подмена изотопов в теле может сделать человека более устойчивым к радиации. Типа кальций, углерод, воду - всё заменить более тяжелыми стабильным изотопами и тогда излучение из космоса снизит урон, так как не сможет разбивать атомы так легко, повреждая днк и другие мелкие структуры.
Аноним 08/06/19 Суб 19:10:13 488192369
>>488189
Тут не знаю, но вангую, что это хуйня. Насколько я понимаю, самыми уязвимыми для радиации будут легкие атомы, то есть водород. А вот его заменять, по крайней мере в значительных количествах, не стоит. Пушо как раз дейтерий слабоядовит. Может, процентов 10 и можно заменить, но вряд ли больше, иначе бо-бо. Так или иначе, наверняка там речь идет не об охуенной защите, а "можно сделать человека на 3% устойчивее к радиации".
Аноним 08/06/19 Суб 19:14:54 488194370
У меня вопрос к оп-посту.
Что значит вопрос:

Q: Можно быстрее?
A: Можно упасть в пузырь альбукерке, наса уже почти надула его.

Немного упорот, извините.

Заранее спасибо!
Аноним 08/06/19 Суб 19:20:38 488195371
>>488194
Там пузырь Алькубьерре подразумевается.

Вкрации: по современным пердставлениям, двигаться в пространстве быстрее скорости света в вакууме нельзя. Но нет ограничений (или, скорее, они просто неизвестны) на то, как может корежиться пространство. Идея в том, чтобы сделать "следующую точку" твоего маршрута ближе, нужным образом "подкорректировав" пространство. То есть ты его "перед собой" скукоживаешь, продвигаешься по этой скукоженной части, а потом раскукоживаешь ее обратно до оригинальных размеров. Такой себе варп-движок в научной формулировке.
Аноним 08/06/19 Суб 19:23:10 488196372
Аноним 08/06/19 Суб 19:30:34 488197373
>>488195
Так а шо там у насы?
Аноним 08/06/19 Суб 19:47:29 488198374
>>488192
>дейтерий слабоядовит
да, в больших концентрациях. Я давно читал статью, но там речь идет о продолжительном обогащении организма сверхмалыми дозами, пока он не начнёт заменять значительный процент в организме.
Аноним 08/06/19 Суб 19:54:00 488199375
>>488198
Так дейтерий как раз ядовит если его в организме дохуя, без разницы, накопилось так или магически сразу все атомы заменились. Ты можешь без особых последствий хоть пол-литра тяжелой воды ебануть, главное чтобы однократно. А вот если так каждый день делать, то начнутся проблемы. А если каждый день пить по наперстку (и 2-3 литра обычной воды, как всегда), то он и накапливаться не будет. Ты его выссышь точно так же, как обычную воду.
Аноним 08/06/19 Суб 19:57:27 488201376
>>488197
Не слежу за новостями, но вангую, что ровно нихуя. Это из разряда "упражнений для ума", судя по всему.
Аноним 08/06/19 Суб 21:10:01 488211377
>>488199
Ок, я тебе тогда ссылку на статью кину, может я не так перевел её что-ли
http://www.oncotarget.com/index.php?journal=oncotarget&page=article&op=view&path%5b%5d=24461&path%5b%5d=76813

В параграфе Deuterated organic compounds говорится про токсичность, но также замечается что это имеет очень сильный анти-раковый эффект и может увеличивать долголетие в определенных случаях. Может дело в том как это балансировать, можно в периоды риска вспышек подменять воду на тяжелую воду в рационе питания космонавтов.
Аноним 08/06/19 Суб 21:32:33 488217378
>>488211
>Conclusions
>...While many of the strategies proposed above may seem speculative, they should be considered as a foundation for future research directions.
Короче, по сути ничего практически полезного (с нынешними технологиями) тут нет.

Хотя там есть то, о чем я не подумал: радиация приводит не только к непосредственным повреждениям в той же ДНК, но и может вызывать ионизацию всяких кислородов, которые уже потом чисто химически ебут все вокруг. И дейтерий более полезен как защита от этого, потому что связи D-C крепче, чем H-C (с протием, то бишь).

Но один хуй, непонятно как ввести в организм достаточно дейтерия чтобы его с одной стороны не убить кхуям, и с другой стороны так, чтобы в ДНК его стало МНОГО. ДНК ведь генерируются не централизованно, а при размножении клеток, то есть по всей тушке. А если ты не можешь разделить дейтерий "по назначениям", то в ДНК его будет столько же, сколько во всем организме в целом, плюс-минус. То есть максимум процентов 10-20 от всех атомов водорода, иначе пизда котенку. Короче, я все равно не вижу, как это может качественно улучшить радиозащиту, разве что количественно и не в разы.
Аноним 08/06/19 Суб 21:38:02 488218379
>>488217
Ну так там и про углерод-13 говорится. Ты видать прост проф деформировался и думаешь, что чтение конлюжн является заменой чтению статьи. В то время как в статье идут широкие намеки на то, что практически полезного навалом, но нужно продолжать исследования. И эти исследования могут сэкономить метры защиты от радиации. Гораздо удобнее защитить человека от радиации изнутри, также как и защитить от деградации костной ткани изнутри, а не при помощи центрифуг ебаных.
Аноним 09/06/19 Вск 01:26:21 488241380
>>488146
>аведенная радиоактивность.
И какие там были изотопы. Насколько я знаю, наведенной радиоактивности подвержены далеко не все материалы и графит с водой в реакторах и используют потому что они слабо "набирают" радиацию. Графит, мне думается, от контакта с радиоактивными материалами со временем просто ими пропитывается, радиоактивными изотопами стронция, цезия, йода и хуй знает чем еще, в продуктах деления урана чего только нет.
Аноним 09/06/19 Вск 01:30:10 488244381
>>488147
Растения слабо подвержены действию радиации, в среднем смертельная доза для растений в несколько тысяч раз выше чем для человека.
Аноним 09/06/19 Вск 11:35:20 488271382
pepe.png (106Кб, 740x982)
740x982
На какую максимально орбиту солнца можно пихнуть зеркало/линзу и сколько там будет ватт а метр?


Роскосый сможет в недосферу дайсона? можно ли концентрированное световое излучение в тыщи терраватт с помощью приемника на L1 переконвертировать с потерями в какой нибудь микроволновое излучение что бы безопасно передавать на землю?
Аноним 09/06/19 Вск 11:38:08 488273383
Нахуя? Тут термояд на подходе.
Аноним 09/06/19 Вск 11:38:51 488274384
>>488201
Я чет слышал вроде, что какие-то небольшие успехи у них есть.
Аноним 09/06/19 Вск 11:43:16 488275385
>>488273


Нету денег на столько термояда, ИТЭР стоит 15 лярдов за 1000 мвт тепловой энергии. Несколько сот зеркал 100 на 100 метров выйдут на круг в 100 раз дешевле.


Для того, что бы перевооружить только текущую экономику, нужно построить кабанчиком более 5000 АЭС, это 60 триллионов долларов, А термояд стоит буквально дороже в 10 раз. Недосфера дайсона будет стоить максимум с десяток МКС.

Даже на солнечные панельки перевооружить нету ресурсов, серебру придет крышка в 2030.
Аноним 09/06/19 Вск 12:17:06 488279386
>>488271
Дохуя таких проектов было, все сыпались именно на беспроводной передаче энергии.
Аноним 09/06/19 Вск 12:25:24 488280387
>>488141
Потому что сильное взаимодействие.
Аноним 09/06/19 Вск 13:28:32 488286388
>>488275

Можно просто чуть-чуть пустыни застроить солнечными батареями/тепловыми генераторами. Никаких сложных космических технологий.

> Даже на солнечные панельки перевооружить нету ресурсов, серебру придет крышка в 2030.

О, это штото новенькое. Раньше /зоговцы и пидорашки обычно пугали заканчиванием редкоземельных элементов.
Аноним 09/06/19 Вск 13:32:48 488290389
>>488286

Нету столько серебра для 70 террават панелей
Аноним 09/06/19 Вск 14:07:57 488307390
>>488290
>Нету столько серебра для 70 террават панелей

а) Ты спутал "разведанные запасы серебра которые рентабельно добывать по текущей цене" c "нету".
б) есть заменители. менее эффективные но и технологии не стоят на месте.

Ну и если действительно будет все плохо солнечные паровые турбины/машины /двигатели стирлинга нам в помощь.
Аноним 09/06/19 Вск 15:18:52 488323391
>>488275
>нужно построить кабанчиком более 5000 АЭС
Экономисты уровня /b.
Аноним 09/06/19 Вск 16:20:42 488335392
>>488323


Ты знаешь энергобаланс угля, нефти и газа? он колоссален, мы говорим не только про электричество, но и про транспорт, промышленность, строительство, дофига всего.

+ что бы заметно поднять уровень жизни на планете нужно в разы больше добычи и производства даже с учетом растущей энергетической эффективности.


Единственный доступный по цене путь, это отправить на близкую солнечную орбиту группу зеркал и линз, что бы прислать сюда концентрированный солнечный зайчик в тысячи тераватт, а строить тысячи термоядерных электростанций которые в 10 раз дороже атомных это чистое безумие, я не понимаю, зачем вообще на это даже деньги тратят, цена термояда всегда будет запредельно космической, достаточно пролистать количество и качество оборудования на сайте ИТЭР, это чистое безумие.

Так же с ВИЭ, что бы закрыть текущий энергобаланс нужно наштопать панелей размером с 70% Испании а промышленность должна быть уже сейчас в 37 раз больше чем сегодня, что бы выдать это хотя бы к концу века, и стоить это все будет просто астрономические деньги.
Аноним 09/06/19 Вск 16:36:53 488340393
да
Аноним 09/06/19 Вск 16:52:14 488342394
image.png (637Кб, 600x600)
600x600
>>488335
>ИТЭР
>Экспериментальный
Щас бы по экспериментальным прототипам судить о серийных.
Аноним 09/06/19 Вск 16:54:55 488343395
>>488273
>>488335
>стоить это все будет просто астрономические деньги
Ага, ну а "группу зеркал" ты тип почти бесплатно отправишь, ну да.

Каждое надо будет снабдить двигателем, ведь они будут быстро вращаться вокруг Солнца, значит угол с Землей будет постоянно меняться. Поскольку они будут близко к Солнцу, температуры там будут адские. Несмотря на это, материал зеркал не должен подвергаться температурной деформации (иначе нифига не на приемник они отражать будут). И двигатели тоже должны работать бесперебойно.

Приемник должен не расплавиться от гигантского количества проходящей энергии. То есть, либо он должен быть коллосальных размеров, либо их должно быть много. Ну или ты собрался светить прямо на Землю, со всеми вытекающими последствиями для экологии, а также с лучами смерти при любом сбое "прицеливания", выжигающими города.
Аноним 09/06/19 Вск 16:59:36 488345396
>>488342


Даже если он выйдет на цену ВВЭР-1200 магически за меньше чем 10 лет, то нужно будет строить тысячами, если не десятками тыщ.


Пару сотен запусков сверхтяжей с полимерными линзами и зеркалами как то дешевле будет на порядок.
Аноним 09/06/19 Вск 17:01:46 488346397
>>488345
Миллиардами, чо уж.
Аноним 09/06/19 Вск 17:06:25 488347398
>>488343


>Каждое надо будет снабдить двигателем, ведь они будут быстро вращаться вокруг Солнца, значит угол с Землей будет постоянно меняться.

Зачем, сами зеркала будут неподвижными, просто будет импульсная система, зайчик будет налетать на приемник каждый час или даже каждые 10 минут(скорость вращения близко к солнцу очень высокая) это будут просто тупые отражающие куски, естественно никакие рули и автоматика не выдержит и год на тех орбитах.


Вот запил приемника(ков), это да, но в целом если такой массив энергии можно будет переработать в безопасные волны где нибудь на L1 и спускать дальше на землю, даже с огромными потерями, то будет огромная экономия по ёмкости материалов и цене если сравнивать с термоядом которого ещё не будет лет 5(если опять не отодвинут) или панельками и батареями.

Что, бы солнечные панели заменили уголь и газ хотя бы в электричестве на 60% к 2050 году, скорость их развертывания с 2015 года должна была быть в 6 раз быстрее чем сегодня.
Аноним 09/06/19 Вск 17:08:05 488348399
>>488346


Тебе сложно посмотреть на википедию где написано число реакторов, какой процент они занимают в генерации электроэнергии и какой процент сама электроэнергия занимает в энергобалансе земли?

ты идиот?
Аноним 09/06/19 Вск 17:14:22 488350400
>>488347
>(скорость вращения близко к солнцу очень высокая)
Все ясно, ты - неадекват. (если хочешь размещать зеркала на низкой солнечной орбите на существующих технологиях и знаниях)

> это будут просто тупые отражающие куски
Это невозможно. Без систем ориентации там все быстро собьется - в том числе от солнечного ветра, светового давления, неравномерностепй гравитации и так далее.

> Что, бы солнечные панели заменили уголь и газ хотя бы в электричестве на 60% к 2050 году, скорость их развертывания с 2015 года должна была быть в 6 раз быстрее чем сегодня.

Ты забываешь ветряки которые являются сейчас основной альтернативкой (сюрприз)

Но это по сути не важно - гораздо проще нарастить производственные мощности известной технологии в 6 раз чем запилить систему к которой 95% технологий просто нет в принципе и неизвестно когда будет. Например что бы чтото хотя бы на орбиту меркурия пульнуть нужно столько дохуя дельты что там для мелкозондлов нужен либо свертяж либо много гравиманевров.
Аноним 09/06/19 Вск 17:19:52 488353401
>>488350


>Все ясно, ты - неадекват. (если хочешь размещать зеркала на низкой солнечной орбите на существующих технологиях и знаниях)



Это возможно, и гравитацию с ветром можно просчитать, даже в целом и автоматика возможна, просто защита нужна будет тяжелая, но зеркало, даже очень маленькое на очень нзкой солнечной орбите сможет собрать сотни или даже тыщи тераватт,

Цена на ракету не проблема, двигатель Raptor готовят к 20 долларам за килоньютон например(50 ре-юзов) вполне себе копеешно получается если так прикинуть.
Аноним 09/06/19 Вск 17:20:47 488354402
>>488348
Просто иди нахуй, прожектер.
Аноним 09/06/19 Вск 17:25:03 488355403
>>488354

>маньке припекает от факта, что экономика существует.

Ясно.
Аноним 09/06/19 Вск 17:27:24 488356404
>>488347
>импульсная система
Хуимпульсная. Нахуя вообще твоя ебала тогда нужна, если она работает одну секунду в три дня? Или ты собрался аккумулятор еще вкрячить в свое зеркало?

Я, кстати, вообще не уверен, что возможно, по крайней мере с нынешними технологиями, создать зеркало, хотя бы статическое, которое светит ровно в заданную точку за сотню миллионов километров.
Аноним 09/06/19 Вск 17:28:57 488357405
>>488355
Даун настаивает на своем утопическом прожекте, хотя ему уже стопицот раз пояснили, почему это невозможно в ближайшие сто лет как минимум. Пиздуй нахуй короче, школьник.
Аноним 09/06/19 Вск 17:44:25 488362406
>>488356


>Хуимпульсная. Нахуя вообще твоя ебала тогда нужна, если она работает одну секунду в три дня? Или ты собрался аккумулятор еще вкрячить в свое зеркало?

Так их там будет много, несколько тыщ на одной и той же орбите, буфером будут бочки с солью на земле или ещё какая нибудь ебала размером со стадионы, несколько тыщ таких по всему шарику.
Аноним 09/06/19 Вск 19:02:00 488368407
1486552740421.jpg (83Кб, 1080x1080)
1080x1080
Хочу написать сцай-фай рассказ, но нужно чтобы было как-то минимум твёрдо и чтобы там были межзвездные путешествия. Огромный корабль с двигателям на металлическом водороде, при пролёте мимо звезды назначения делает маневр у планеты гиганта и сбрасывает пассажирский модуль, который тормозят лазерами с орбиты луны газового-гиганта. Сам межзвездный корабль продолжает путешествие к другой звезде без торможения.

Есть ли дыры в такой схеме, на которые потом будут плевать, как на нетвердоту?
Аноним 09/06/19 Вск 19:08:43 488369408
>>488368
>делает маневр у планеты гиганта
>продолжает путешествие к другой звезде без торможения
Если он не тормозит с межзвездных скоростей, то любой маневр будет пуком, он, считай, ни на что не повлияет.

Ну и плюс на таких скоростях любой мимокамушек (которых в звездных системах полно) станет мегабомбой. Хотя тут можно сослаться на неведомую йоба-защиту. В конце концов, за пределами звездных систем тоже можно об камушек споткнуться, так что какая-то защита один хуй желательна.
Аноним 09/06/19 Вск 19:13:31 488370409
Если земляне построят звезду смерти, которую будет можно наблюдать с земли не намного хуже луны, она будет влиять на приливы-отливы, лунатиков, оборотней итд.?
Аноним 09/06/19 Вск 19:24:05 488371410
>>488369
Ок, напишу, что лазером на пути следования корабля всё сжигают нахуй, чтобы ни одного камешка. Маневр у гиганта нужен лишь чтобы лазер не был каким-то уж совсем йоба стреляющим на полсистемы. На луне-гиганта стоит много электростанций, которые питают лазер, убивают скорость капсулы с экипажем. Капсулу подбирает корабль-буксир и тащит куда нужно.
Аноним 09/06/19 Вск 19:42:00 488372411
>>488370
>она будет влиять на приливы-отливы
Да. Примерный перепад гравитации от тела на противоположных точках Земли равен G m (1/(R-r)^2 - 1/(R+r)^2) = G m 4 Rr / (R-r)^2 (R+r)^2, где R - расстояние от центра Земли до тела, r - радиус Земли, m - масса тела, G - константа. Пусть твоя ЗС в N раз меньше Луны по линейным размерам, но видна под тем же углом. Для простоты возьмем плотность равной Луне. Тогда перепад гравитации будет (L для Луны) равен: G (mL / N^3) (RL / N) r / (RL/N-r)^2 (RL/N+r)^2. Если R >> r видно, что и в числитель и в знаменатель N входит в -4 степени и "сократится". (Там настоящего сокращения быть не может, но порядок один).

На пальцах: если ЗС в 10 раз линейно меньше Луны, но в десять раз ближе, она будет тянуть в 10 раз слабее, но за счет того, что отношение "расстояние до объекта / радиус Земли" станет в 10 раз меньше, перепад гравитации будет примерно такой же. Аналогично если она в 10 раз больше Луны, но в десять раз дальше.

Короче, приливы-отливы будут примерно такие же как от Луны, правда с поправкой на плотность твоей ебалы.
Аноним 09/06/19 Вск 20:11:40 488376412
Аноним 10/06/19 Пнд 02:16:04 488404413
>>488372
>>488376
Кстати, я немного ошибся. Для такого же видимого угла расстояние будет чуток больше, потому что в N раз уменьшится расстояние до поверхности Земли, а не до ее центра. Но принципиально это на ответ не повлияет, может приливы и будут, скажем, на 50% меньше (при N = 10), но по крайней мере того же порядка. Ну и чем ближе N к единице, тем меньше ошибка.

Алсо, пока никто не доебался: камушек в вытянутой руке не дает таких приливов как Луна по двум причинам: одна указана выше, а вторая в оригинальном посте: приближение работает только при R >> r, то есть надо хотя бы R = 3r.
Аноним 11/06/19 Втр 11:07:41 488545414
Почему пельмени всплывают при варке?
Аноним 11/06/19 Втр 12:05:57 488553415
>>488545
Надуваются паром и всплывают. Если дать остыть вместе с бульоном - утонут нахой, т.к. пар сконденсируется и пельмешы уменьшутся в объёме, их плавучесть снизится.
Аноним 11/06/19 Втр 12:34:56 488564416
>>488553
Какое давление поддерживают на космических станциях? Одна атмосфера или меньше? Можно ли там сварить пельмени в кастрюльке-центрифуге или нужна скороварка?
Аноним 11/06/19 Втр 13:35:05 488569417
>>488564
> Одна атмосфера или меньше?
это
>Можно ли там сварить пельмени в кастрюльке-центрифуге или нужна скороварка?
можно без центрифуги, только крышку плотнее прижимай и вытяжку включить не забудь, парует сильно
Аноним 11/06/19 Втр 13:44:05 488571418
>>488569
>это
Так одна? Или 0,7, скажем, для экономии?
>только крышку плотнее прижимай
Это уже скороварка будет. Имеется в виду, достаточно ли давления на МКС для традиционной варки в обычной кастрюле если просто создать эрзац-гравитацию, или нужно еще дополнительное давление в скороварке? Дома я могу пельмени в кастрюльке сварить. А на вершине Эвереста, например, вода кипит при 70 градусах, этого вроде как мало для нормальной варки.
Аноним 11/06/19 Втр 14:19:11 488578419
>>488571
> Так одна? Или 0,7, скажем, для экономии?
одна, раньше бывало заливали меньше, но чистого кислода
> Это уже скороварка будет. Имеется в виду, достаточно ли давления на МКС для традиционной варки в обычной кастрюле если просто создать эрзац-гравитацию, или нужно еще дополнительное давление в скороварке? Дома я могу пельмени в кастрюльке сварить. А на вершине Эвереста, например, вода кипит при 70 градусах, этого вроде как мало для нормальной варки
посыл ясен, довления хватит, но космонавтикам будет ниоч от варки
объём-то закрытый, бздунов не берут, потеть только на тренажорах, вся хуйня
тебя с твоей кастрюлькой недопоймут
Аноним 11/06/19 Втр 14:31:55 488582420
>>488578
Спасибо. Так-то понятно, что никто там пельмешки варить не будет, мне просто интересна принципиальная возможность.
Аноним 11/06/19 Втр 18:53:03 488607421
Сколько космонавт нахватается радиации за полет на Марс? Лучше свинца и йода ничего не придумали, а лететь от 4 до 7 месяцев в зависимости от расположения планет.
Читал, что даже на МКС нихуевая доза радиации за год пребывания, а они там под магнитным пледиком Земли
Аноним 11/06/19 Втр 19:19:40 488608422
IMG201906111918[...].jpg (223Кб, 900x1017)
900x1017
Аноним 11/06/19 Втр 19:23:52 488611423
IMG201906111922[...].jpg (238Кб, 900x1017)
900x1017
Аноним 11/06/19 Втр 19:28:58 488614424
>>488607
> Лучше свинца и йода ничего не придумали
так-то альфу похуй чем гасить лишь бы масса
Аноним 11/06/19 Втр 19:30:44 488616425
>>488614
Да я хз какая именно радиация в космосе
Ну и корабль свинцом не обшить, дя
Аноним 11/06/19 Втр 19:33:58 488618426
>>488616
бутерброд между баками с топливом/окислителем/водой/воздухом
платина же
Аноним 11/06/19 Втр 19:39:42 488622427
>>488611
Пиздец масштаб уебищный. Нихуя не понятно сколько получишь за 180 дней полета до Марса
Аноним 11/06/19 Втр 19:42:36 488623428
>>488622
логарифмический, жри давай
Аноним 11/06/19 Втр 19:51:28 488625429
>>487675
>Потому что окислитель
И чо?
>>487704
А диапазон изменения температур ты считал?
>>487876
>если образуется пар, то теплопередача через него сразу резко падает, и стенка может перегреться и прогореть.
Давай, расскажи это паровым котлам.
Аноним 11/06/19 Втр 20:54:40 488630430
>>488625
> Давай, расскажи это паровым котлам.
> рд
> паровые котлы
Аноним 11/06/19 Втр 21:41:19 488633431
Можно ли при нынешних технологиях погулять по Титану?

Скажем, условия такие: титанавта в изготовленном Илоном Маском на заказ скафандре (вес на Земле не более 50 кг, чтобы исключить "скафандры"-танки) магически телепортируют на поверхность Титана, в сухое ровное место. То есть, лететь туда не надо. Он должен пешочком протопать минимум 10 км, и пробыть там минимум 3 часа. После чего он так же магически телепортируется обратно. Выживет? Маску дается три года на подготовку.

Условия на поверхности: температура около -180, плотная атмосфера (в 4 раза плотнее земной), давление около 1,5 атм. Ветер слабый, не более 0,5 м/с. Возможна моросня их жидкого метана, но дождя не будет.
Аноним 11/06/19 Втр 21:51:18 488635432
>>488633
>вес на Земле не более 50 кг
ты охуел, даже скафандры для космоса и луны больше весят, а там вакуум и теплопередача крайне мала
Аноним 11/06/19 Втр 22:21:27 488639433
>>488635
Ну хорошо, пусть 100 кг.
Аноним 11/06/19 Втр 22:32:01 488642434
>>488639
Ладно, я сегодня добрый. Пусть 105 вес любой, лишь бы титанавт в нем самостоятельно ходить мог (моторы и прочие экзоскелеты недопускаются). Впрочем, сила тяжести на Титане около 1/7 земной, это должно помочь.
Аноним 12/06/19 Срд 16:21:06 488750435
>>488241
C-14. А продукты деления урана находятся внутри оболочек ТВЭЛов. Даже при их разрушении и попадании продуктов деления в КМПЦ, они не осядут на графит, потому что СУЗ охлаждаются своей отдельной петлей.
Аноним 12/06/19 Срд 16:57:06 488762436
Теоретически, что будет, если Ио упадёт на Юпитер?
Аноним 12/06/19 Срд 17:03:35 488764437
Аноним 12/06/19 Срд 18:53:15 488793438
>>488764
Это понятно, а остальное? Что с юпитером будет и остальной солнечной системой?
Аноним 12/06/19 Срд 19:04:04 488798439
>>488793
похуй вообще
резонанс спутников юпитера немного поменяется и всё
12/06/19 Срд 19:37:39 488814440
>>488630
Продолжай деградировать
Аноним 13/06/19 Чтв 11:51:32 488875441
image.png (898Кб, 900x777)
900x777
Можно ли увеличить дельту пердежом?

Смотрите, у нас есть корабль до марса и астронавты которые пердят. Можно ли собирать метан при рециркуляции воздуха и использовать его как топливо для дополнительного малого двигателя.
один человек выпускает в день до 0.5 литров газов из которых четверть это метан. То есть примерно 0.125. Четыре астронавта напердят 0.5 метана в день. Если полет продлится 100 дней в одну сторону, то это 50 литров метана. В обе стороны 100 литров. 100 литров это ровно одна бочка.

Внимание вопрос - сколько дельты можно получить из бочки сгоревшего метана?
Аноним 13/06/19 Чтв 12:13:08 488878442
>>488875
Вопрос не имеет смысла без массы корабля, но в любом случае даже метр в секунду вряд ли наберется.
Аноним 13/06/19 Чтв 12:27:53 488879443
>>488875
>из которых четверть это метан
Что-то много. Там же вроде чуть ли не все - воздух, просто побольше азота, поменьше кислорода.
Аноним 13/06/19 Чтв 12:34:22 488880444
image.jpeg (136Кб, 476x497)
476x497
Платина, наверное.

Поясните за то, как из сверхновой звезды образуется черная дыра.
Ведь при взрыве сверхновой идет выброс массы из ядра наружу, то есть, сверхновая теряет массу, а потом образует черную дыру, которая за счет своей большой массы притягивает все окружающие объекты.

Сча поясню, почему я не понимаю, как из сверхновой образуется ЧД.
Предположим, есть масса Nчд, набирая которую объект получает огромную силу притяжения и становится ЧД.

Но как сверхновая (тут ее масса меньше необходимой Nчд) теряя массу, выбрасывая ее из ядра, становится черной дырой?
Поясните, где я не прав.
Я далек от астрономии, просто увидел ролик на ютубе и решил спросить у вас.
Аноним 13/06/19 Чтв 12:43:30 488881445
>>488880
>Предположим, есть масса Nчд, набирая которую объект получает огромную силу притяжения и становится ЧД.
Лучше не предполагай, это неверно. ЧД может быть любой массы.

ЧД, которая получается из звезды, всегда заметно легче самой звезды, потому что в процессе значительная часть массы разбрасывается взрывом. Масса сворачивается в ЧД не столько потому, что ее много, сколько потому что ее много в малом объеме.

Любое количество вещества стремится занять "нулевой" объем под действием самогравитации. Но ему мешают всяческие силы. В звезде, например, это давление разогретого газа. Вот когда эти силы становятся слишком слабыми в силу разных причин, и не могут компенсировать самогравитацию, вещество коллапсирует и получается ЧД. Другой вариант - вещество сжимают внешними силами, но это не про звезды.
Аноним 13/06/19 Чтв 12:52:44 488883446
>>488881
Теперь понятно, что дело в объеме. Совсем не подумал об этом. Спасибо.

> занять "нулевой" объем
Это очень маленькое межатомное расстояние?
Аноним 13/06/19 Чтв 12:56:18 488884447
>>488883
И даже дальше. В нейтронной звезде, например, уже нет атомов (кроме, может, тонкой внешней "корочки"), их "давит" самогравитацией. Но и там еще есть силы, которые не дают сжаться еще больше - это уже чисто квантовомеханические эффекты.
Аноним 13/06/19 Чтв 13:29:12 488887448
>>488883
Это буквально нулевой, т.е. точка или кольцо нулевой толщины. В сингулярности все элементарные частицы занимают одно и то же положение в пространстве, потому что не существует никаких сил, способных их друг от друга оттолкнуть.
Аноним 13/06/19 Чтв 14:09:17 488888449
Можно ли создать такие бактерии, которые бы хорошо выживали на марсе (экватор), питались бы пылью и чем попало вокруг, атмосферой, солнечной радиацией, и в ходе жизнедеятельности выделяли кислород?
Если заселить из на марсе и дать расти 100 лет, то могут ли они сделать его пригодным для жизни.

Заранее спасибо. Наверняка это невозможно, хотелось бы узнать почему.
Аноним 13/06/19 Чтв 14:17:38 488889450
>>488888
Всей земной жизни нужна жидкая вода, на Марсе ее нет (1-2 крошечных полоски, даже не факт, что являющихся водой — не в счет).

Все остальные факторы типа холода, низкого давления, недостатка химических источников питания, и т.д. сами по себе не непреодолимы, но без жидкой воды жизни 100% не будет.
Аноним 13/06/19 Чтв 14:32:16 488890451
>>488887
>не существует никаких сил, способных их друг от друга оттолкнуть
Никаких известных. Хуй его знает, что там происходит на самом деле. Теоретически предсказать нельзя, ведь ОТО и квантмех не стыкуются, можно лишь делать более-менее обоснованные предположения. А экспериментальных данных тоже нет, поскольку необходимые энергии пока недостижимы, а уж экспериментировать с экстремальной гравитацией вообще непонятно как.
Аноним 14/06/19 Птн 14:49:02 488985452
Снимок экрана 2[...].png (534Кб, 916x744)
916x744
Снимок экрана 2[...].png (568Кб, 916x745)
916x745
Снимок экрана 2[...].png (720Кб, 918x744)
918x744
Снимок экрана 2[...].png (908Кб, 909x746)
909x746
Чому больше никто не делает астрономические зарисовки. Ведь это так няшно.
Аноним 14/06/19 Птн 23:17:34 489036453
Предположим, нам дали пульт управления солнечной системой. И мы без задней мысли немного подкорректировали орбиты всякой мелкой шушеры и ебанули по Марсу десятью тысячами комет диаметром около 10 км каждая, в основном из льда (75% и больше) в течение 10 лет. Насколько это повлияет на Марс: атмосферу, поверхность? Сколько понадобиться времени, чтобы он "успокоился" после такого и там можно было выращивать картофель высаживать экспедиции?
Аноним 14/06/19 Птн 23:42:56 489040454
>>489036
А если у тебя пульт управления солнечной системой есть, то хули не передвинуть Марс поближе к Солнцу?

Вообще ну ебнул ты кометами, они создали кратеры, газ высвободился, наполнил атмосферу. Знаешь что дальше будет с этим газом? Газ этот сделает финт ушами и пойдет на полярную шапку, осядет там, сделав её толще размером. И как ты ему запретишь? Скажешь - "Эй, газ, а ну стоят на месте, не двигаться и не замерзать, ты должен быть газообразным, не переходи в твёрдое состояние!!!"
Думаешь он тебя будет слушаться? Даже если ты там как ЭЛОН Маск ебнешь ядерными бомбами по полярным шапкам, ты думаешь газ скажет "О нет, полюса разбомбили, я здесь больше не могу затвердевать, место радиоактивное!" Лол, нет. Газ растает, поциркулирует и снова осядет на полярные шапки, как ни в чём не бывало. Что ему помешает блядь?
Аноним 14/06/19 Птн 23:49:48 489042455
>>489040
Херово. Для парникового эффекта от водяного пара, как я понимаю, жалких 10 тысяч комет недостаточно будет, да? А можно ли в принципе Марс разогреть таким образом (хули, будем считать запас комет бесконечным)? Есть же еще небольшой единовременный разогрев просто от падающих камней, а потом держать полученный профит парником. Двигать орбиту Марса владелец пульта не разрешил, только кометами кидаться.
Аноним 15/06/19 Суб 00:03:20 489044456
Аноним 15/06/19 Суб 00:06:53 489045457
>>489044
Спасибо, но 70 минут как-то многовато для меня.
Аноним 15/06/19 Суб 00:11:30 489046458
>>489042
Там многое зависит от размера комет. Магически ты конечно можешь двигать 10 км кометы, но если бы дело зашло до реальных дел, то тут мы понимаем, что даже 10 метровый камешек на околоземной орбите двинуть для нас невозможно. А уж комета с размерами 10 км и вытянутой орбитой это просто невозможно. Ты там банально охуеешь просто догнать её, а уж чтобы повлиять на эту глыбу это нужно - ну я не знаю, тут просто нереально, чувак тебе просто сложно представить сколько весят эти 10 км камня и какие скорости необходимо применить чтобы изменить орбиту этого колосса. В видео Егоров говорит о том, что нужно под 600 комет размерами с комету Хейлла-Боппа(50-60 км) для того чтобы давление на Марсе стало 1% от земного. В общем просто какие-то диких размеров кометы нужно найти, потом как-то их двигать. 50 км камень прилетающий из Облака Оорта, ему вообще наверное похуй что ты ему не делай, он не заметит даже если ты несколько ракет Сатурн-5 к нему приделаешь.
Аноним 15/06/19 Суб 00:20:43 489049459
>>489046
Не, я понимаю, что это нереально, отсюда и магический пульт. 50-60 км это где-то в 150-200 раз больше чем 10 км по объему. То есть 600 таких будут примерно в 10 раз существеннее 10 тысяч камушков из вопроса. Другое дело, что даже 1% давления (что уже ~100 000 десятикилометровых комет) это ни о чем, да, никакого парника там не получится.
Аноним 15/06/19 Суб 10:47:16 489084460
>>489036
Венеробояре ссут в ротеш марсохолопам итт.
Аноним 15/06/19 Суб 11:38:56 489085461
>>489036
Universe sandbox2 тебе в помощь.
Аноним 15/06/19 Суб 14:17:22 489096462
image.jpeg (197Кб, 604x604)
604x604
>>487150
> На таких телах и гравитация будет мизерной, пукнешь и улетишь нахуй.
Аноним 15/06/19 Суб 16:07:47 489106463
>>489036
высаживать картоху и сажать экспедиции можно хоть сейчас, все условия прекрасно привозятся с землюхи, один хуй ты везти что-то я смотрю собираешься
бомбить итт совершенно не зачем, один хуй приличной атмосферы не сделаешь, а красивые виды цвета разноцветного поноса издырявишь
алсо этого >>489040 не слушай, на марсе уже мульёны лет потепление хуярит, так что не всё в шапки уйдёт, скорее именно полярные головные уборы съебнутся быстро и решительна
Аноним 15/06/19 Суб 20:17:21 489122464
Насколько жоская в космосе радиация? Интересуют все места: около звезд, всяких разных планет, подальше от всего фонящего. Как там вообще в межзвездном пространстве, чего ожидает современная наука? А в межгалактическом? Какой там тип радиации (альфа-бета-гамма, или какие еще частицы бывают)?
Аноним 15/06/19 Суб 23:12:14 489152465
>>489122
Гамма лучи, бета излучения и альфа волны. Пиздецома, гроб , кладбище, рогозин
Аноним 16/06/19 Вск 01:17:53 489164466
88989065873299.jpg (29Кб, 550x412)
550x412
>>489122
В радиационных поясах планет носятся ионы и электроны по магнитным линиям, это по сути естественный ускоритель частиц. Бахаются о корабль, проникают на некую глубину, вызывают гамму. Радиационные пояса у Юпитера с его мощной магнитосферы имеют такую силу, что проще в ядро реактора сразу залезть.

В межпланетном пространстве тоже летают заряженные частицы - солнце хуячит высокоэнергетичными протонами, во время вспышек особенно. Кроме того, небесный рандом иногда присылает ТЗЧ - тяжелые заряженные частицы, которые ебашат с такой силой, что пробивают десять с половиной аллахов навылет, порождая в стороны дождь из других заряженных частиц по траектории прохождения сквозь материал. От таких защиты нет в принципе.

>Как там вообще в межзвездном пространстве
Примерно как в межпланетном, только попроще, нет солнечного ветра.
Аноним 16/06/19 Вск 02:12:55 489170467
Как так Аполлоны успешно летали? Такая сложная миссия, столько этапов. И все без ошибок и аварий(13 не считаем).
Так все предусмотрели и рассчитали?
Аноним 16/06/19 Вск 04:11:28 489174468
>>489170
Так аварии, ошибки, сбои были, дохуя, просто там не такие серъезные. Ну погорели чутка на Аполлон-1. Зато потом были почти никем не вспоминаемые Аполлон-2, 3, 4, 5, 6, 7, 8, 9, 10 во время которых тестили все системы и улучшали, искореняли ошибки. Просто история программы Аполлон это очень длинная история, это как биография человека, которую всем лень читать. Она очень подробна, но сука пиздец людям как лень, не хотят и всё тут. Просто зацикливаются на Аполлон-11 и моменте посадки, а всё что было до, и что было после - все это люди просто игнорируют. Ну и плюс люди не любят читать технические детали миссий, это же тебе не Властелин Колец, не Звездные Войны под попкорн жрать, быдлу это вообще не интересно, там аббревиатуры, фамилии, цифры часто много слишком всего непонятного.
Аноним 16/06/19 Вск 14:18:12 489208469
>>489164
А ТЗЧ эти из реликтового излучения или нет? В межгалактическом они тоже будут?
Аноним 16/06/19 Вск 16:57:38 489228470
>>489208
> заряженные частицы
> микроволновый фон
учу читать в ттв, спешите видеть
Аноним 16/06/19 Вск 17:16:24 489229471
>>489208
ТЗЧ может быть откуда угодно - где-то на другой звезды вспышка произошла, частицы вылетели и полетели. Нейтронные звезды срут там, квазары из других галактик, ебанутые конфигурации из пар звёзд, экзотические всякие объекты могут посылать частицы во всех направлениях, тонны дерьма в нашей галактике и за пределами
Аноним 16/06/19 Вск 17:16:44 489230472
Аноним 16/06/19 Вск 19:52:36 489248473
изображение.png (372Кб, 1080x859)
1080x859
Сап, беру подержанный телескоп (Долбоеб). Сразу говорю, я до этого оптику никакую не покупал и астрономией интересуюсь скорее на уровне "ОООО ЕБАТЬ, ЗВЕЗДЫ, КРУТО!" и хотел бы просто попробовать вести наблюдения.
Написал продавцу - пока молчит, вот что о телескопе было в объявлении.
На что мне смотреть и обращать внимание при покупке? Инб4 на засранность линз и на причину продажи, а еще на то, как телескоп хранился
Аноним 16/06/19 Вск 20:11:40 489253474
Яйцеголовые, почему чистая вода не прозрачная? Почему воду видно в воздушной среде? Почему я вижу пар в виде дыма?
Аноним 16/06/19 Вск 20:37:12 489260475
>>489228
Но ведь у нас квантово-волновой дуализм.
Аноним 16/06/19 Вск 20:38:53 489261476
151446880316642[...].jpg (20Кб, 473x269)
473x269
>>489260
>Но ведь у нас квантово-волновой дуализм.
Чево блядь?
Аноним 16/06/19 Вск 20:41:25 489263477
>>489253
>почему чистая вода не прозрачная?
А она должна быть? У неё свой параметр преломления.
Аноним 16/06/19 Вск 20:42:20 489264478
>>489261
Корпускулярно-волновой, простити.
Аноним 16/06/19 Вск 22:31:24 489281479
>>489263
>преломления
Прозрачность. Именно как физическая характеристика. Коэф. преломления это ответ на второй вопрос. А вообще это спейсач.
>>489264
>Корпускулярно-волновой
И к чему ты это. Значение знаешь? Пит буль.
Аноним 16/06/19 Вск 22:34:25 489282480
>>489281
Ну тип все частицы в какой-то степени одновременно и волны. Могу теперь предположить (по твоей реакции), что реликтовое излучение состоит только из фотонов, но я не знал / не задумывался, честное пионерское!
Аноним 16/06/19 Вск 22:45:06 489284481
>>489282
>Ну тип все частицы в какой-то степени одновременно и волны.
Ну тип в начале прошлого века ты был бы прогрессивным, но это уже на той же полке, где и атом Бора. Сейчас частицы уже не принято пытаться описывать упругими шариками, которые иногда волны.
>предположить
Смелее предполагай, как-то неуверенно. Да, из фотонов.
Аноним 16/06/19 Вск 22:52:18 489285482
>>489284
Охуеть ты короче популяризатор науки. Лучше не задавать тупых вопросов в треде тупых вопросов, а то набигут такие буки как ты.
Аноним 16/06/19 Вск 23:03:35 489288483
Аноним 16/06/19 Вск 23:09:35 489289484
>>489285
Ну для старта - не популяризатор, даже не стремящийся.
А как относиться, когда анон приходит с воззрениями, которым уже сто лет (ну пусть 50-60, когда там ктп Фейнман замесил?), и это его текущий уровень. Ну а самому всерьез отвечать, круглая ли Земля, вертится ли, было бы в радость? Я может тоже зашел что-то новое узнать. Лучше бы не дулся, а почитал что-нибудь популяризаторское, видосики там образовательные посмотрел. Это тот уровень, который можно без обсуждений и вопросов воспринимать, а вот немного освоившись в современных представлениях станет интереснее и обсуждать в тредах тупых вопросов нюансы и странности.
Аноним 17/06/19 Пнд 01:34:10 489297485
>>489288
ну ты так-то совсем уж йобу принёс, но в целом верно, в наши спокойные ебеня такой хуйни особо не прилетает, а если и прилетает, то не задерживется
другое дело совать жопу в квазар или дождаться когда по тебе прицельно (или не очень) жахнет лучем или сверхновой
алсо возле особо крупных и магнино-могучих юпов тоже собирается много тяжелого дерьма( пускай и не настолько адового) к юпам лучше не лезть
Аноним 17/06/19 Пнд 01:40:13 489299486
monkey confused[...].jpg (79Кб, 500x499)
500x499
>>489288
> At this speed, if a photon were travelling with the particle, it would take over 215,000 years for the photon to gain a 1 cm lead as seen in Earth's reference frame.
переведите плз
Аноним 17/06/19 Пнд 01:48:35 489300487
>>489299
>На этой скорости, если бы фотон летел рядом с частицей, потребовалось бы 215 000 лет, чтобы фотон обогнал ее на 1 см, в системе отсчета Земли.
Аноним 17/06/19 Пнд 01:49:19 489301488
Аноним 17/06/19 Пнд 09:24:27 489319489
>>489248
На состояние зеркал
Аноним 17/06/19 Пнд 12:08:06 489334490
>>489248
Не в Томске покупаешь?
Аноним 17/06/19 Пнд 12:56:19 489341491
>>488183
Не только из-за вреда для костей.
Если внимательно посмотреть как выглядят астронавты на МКС, то станет заметным совершенно иное распределение давлений в организме в условиях без гравитации: чем ближе к сердцу - тем выше давление, чем дальше от сердца - тем меньше давление и меньше кровоток. Лицо, голова и грудная клетка в невесомости припухшие, ноги и нижняя часть туловиша болкк тонкие и "усохшие" по сравнению с наличием силы тяжести.
Поэтому полного лекарственного решения проблемы не существует, нужно либо вращать шконки вместе с огромным тороидом, либо развивать постоянное ускорение от 0.5g и выше.
Но тороид по сравнению с шаром обладает довольно херовым отношением внешней площади к полезному объёму, внешнюю защиту шара можно сделать более толстой.
Аноним 17/06/19 Пнд 13:44:58 489354492
Аноним 17/06/19 Пнд 14:18:17 489359493
Анон, у меня вдруг появилась мысль, которая теперь не дает мне покоя: что, если бы у Земли было две Луны?
Это сильно ли бы повлияло на геологию, климат, гравитацию, длительность суток и года, физиологию, развитие ночных видов животных и растений, развитие навигации, астрономии, календаря, земледелия?
Что было было?

Для простоты возьмем два тела совокупной массой равной массе настоящей луны, один объект в 40% от ее массы, второй - 60%.

Приветствуется ссылка на литературу по теме, если такая вдруг окажется.
Аноним 17/06/19 Пнд 14:44:15 489363494
>>489341
Можно не делать огромный тороид, а например, сделать противовес на длинном тросе, вынеся тем самым центр вращения за пределы жилого отсека.

А там уж жилой модуль можно любой формы хуячить.
Аноним 17/06/19 Пнд 14:44:54 489364495
А есть ли снимки Луны на фоне Млечного Пути?
Аноним 17/06/19 Пнд 14:52:22 489365496
>>489354
душ из мульёна частиц опаснее одной еболы (при равной энергии) потому как одна частица потеряет меньше энергии( и соответственно меньше передаст в тело космонавтика)
это верно в случае достаточной для преодоления пути до тела энергии
Аноним 17/06/19 Пнд 14:53:07 489366497
>>489364
Только композитные, в один снимок такое никак не сделать — слишком большая разница в яркости.
Аноним 17/06/19 Пнд 14:54:03 489367498
>>489359
Повлияло бы на некоторые вещи тобою описанные и не повлияло бы на другие, но описывать как именно это повлияло бы на 12 аспектов мироустройства при недостаточных данных сложно да и лень вообще. Нужно знать орбиту второй луны.
Аноним 17/06/19 Пнд 14:55:33 489368499
>>489365
Ну как бы частица может попасть в скафандр и за стенкой скафандра сразу же будет этот душ. Или же она может пройти под углом снизу вверх в тело космонавта, попасть в ногу, прореагировать с кожей и душ из частиц начнет распространяться внутри тела.
Аноним 17/06/19 Пнд 14:56:40 489369500
>>489366
> Только композитные
а если снимать когда на луне ночь?
Аноним 17/06/19 Пнд 14:59:37 489370501
А планет на фоне Млечного Пути? Юпитера там или Сатурна
Аноним 17/06/19 Пнд 15:09:44 489371502
gc8y3ga4ja331.jpg (660Кб, 4016x2677)
4016x2677
>>489369
Даже небольшого серпа достаточно для засветки.

>>489370
Ну этого как говна, но они ведь как точки выглядят без сильного увеличения, крутых картинок не выйдет. И все равно разница в яркости настолько большая, что планеты выходят засвеченными добела.
Аноним 17/06/19 Пнд 15:12:12 489373503
>>489367
а какие данные нужны?
Аноним 17/06/19 Пнд 15:13:29 489374504
>>489368
дело в том, что более тяжелые частицы меньше хотят в неупругие столкновения и взаимодействуют главным образом с электронными оболочками
Аноним 17/06/19 Пнд 15:17:06 489375505
>>489374
Всё равно нельзя исключать столкновения - это тебе не нейтрон или нейтрино, из космоса может прилететь ион железа на гигавате чтобы разьебать кому-то сраку.
Аноним 17/06/19 Пнд 15:25:16 489376506
>>489375
половинный пробег ГЭв-ого протона в люминии - полтора метра, так что все высокоебучие говна с КРАЙНЕ ВЫСОКОЙ вероятностью пролетят тельце на вылет практически без последствий для последнего
Аноним 17/06/19 Пнд 15:37:00 489378507
>>489365
То есть 20-50 см воды (грубо говоря, в зависимости от угла попадания) той ебале не хватило бы, чтобы реализовать весь свой "потенциал", и "обломки" соударений все равно вылетели бы на еще огромной скорости?
Аноним 17/06/19 Пнд 15:40:40 489379508
>>489376
Постой, я тут читаю и вижу такую штуку как ионизационные потери, то есть частица высоких энергий может взаимодействовать даже без столкновения, ионизируя атомы по пути следования. Так что даже пролетая навылет через тушку, она может нанести генетический урон, если я правильно понял конечно.
Аноним 17/06/19 Пнд 15:51:36 489383509
>>489378
вообще говоря шансы выпадения "дождя" на таком коротком и неплотном участке крайне малы
и даже в таком случае, ливень не сразу делает из одной частицы мульёны, сперва частица распадётся скажем на 3-5 частиц примерно такой же общей энергии, которые примерно также нихуя не хотят распадаться здесь и сейчас, которые пролетят ещё некоторое расстояние и вероятнее всего покинут обсуждаемое тело космонавтика, толком ни на что не распавшись и особо не тормозя
Аноним 17/06/19 Пнд 15:53:30 489384510
>>489383
Ну да, я примерно это и имел в виду говоря о "потенциале". Типа что есть еще энергия "на поделиться", но уже мишень закончилась, не с чем сталкиваться.
Аноним 17/06/19 Пнд 15:56:37 489385511
>>489379
верно, ты понял правильно, я ведь написал
> взаимодействуют главным образом с электронными оболочками
другое дело что это всё идёт по прямой, имеет околонулевой объем и даёт такие же (никакие) повреждения
Аноним 17/06/19 Пнд 16:04:04 489387512
>>489371
Спасибо, всё равно нормально очень даже. А что это? Юпитер? Спутников не видать? А есть по две планеты?
Аноним 17/06/19 Пнд 16:07:20 489388513
>>489385
>даёт такие же (никакие) повреждения
Нет, это даёт повреждение. В ганимед-треде приводил статью, где риск галактических космических лучей называется главной опасностью при полётах в дальний космос. Причем галактический фон растёт с увеличением расстояния от Солнца и в минимумы солнечной активности.
Аноним 17/06/19 Пнд 16:09:18 489390514
>>489387
Юпитер. Для спутников увеличение побольше нужно, тут все в одно пятно сливается.
Аноним 17/06/19 Пнд 16:58:47 489401515
JamesAlfredVan [...].png (416Кб, 640x618)
640x618
Аноним 17/06/19 Пнд 18:48:31 489418516
>>489388
я тут про хуйни, которые дают ливни, а не про "нормальные" космические лучи распинаюсь если чо
а вот про нормальные галактик рэи всё довольно спекулятивно (в плане точности, сам факт что космические лучи опаснее лучей солнышка и отсутствие приемлимых массогабаритных решений никто не оспаривет), потому как спектр довольно широкий и часть уйдёт в титанолюминий обшивки, часть зайдёт, а часть пролетит
считать конкретные доли и величины ионизационных нагрузок без конкретных шипов - такое себе
Аноним 17/06/19 Пнд 19:18:56 489421517
>>489418
Да считай там пизда от галактических лучей в любом случае - от слабых тебе в любом случае нужно защититься при помощи слоёв защиты, а сильные только сдетонируют от этой защиты и создадут ливень.
Настройки X
Ответить в тред X
15000 [S]
Макс объем: 40Mб, макс кол-во файлов: 4
Кликни/брось файл/ctrl-v
Стикеры X
Избранное / Топ тредов